Anda di halaman 1dari 182

1

Program Cemerlang Bersama SMT Zon Johor Program Cemerlang Bersama SMT Zon Johor Program Cemerlang Bersama SMT Zon Johor Program Cemerlang Bersama SMT Zon Johor
TABLE OF TABLE OF TABLE OF TABLE OF CONTENTS CONTENTS CONTENTS CONTENTS

NO. NO. NO. NO. ITEMS ITEMS ITEMS ITEMS PAGE PAGE PAGE PAGE REMARKS REMARKS REMARKS REMARKS
1. FormuIae 2
2. SPM Paper Format 4
3. Recommended Time AIIocation 5
4. Quick Notes 6
5. Past Years SPM Questions
AnaIysis

Prepared by SMT Pontian
Paper 1 AnaIysis 22
Paper 2 AnaIysis 24
SPM Questions 26
6. ModuIes
ModuIe 1 (paper 1) 69
ModuIe 2 (paper 1) 76
ModuIe 3 (paper 2) 84
ModuIe 4 (paper 2) 89
Answers For ModuIes 96
7. CoIIection of SPM TriaI 2008
SeIangor (P1 & P2) 103
KuaIa Lumpur (P1 & P2) 117
Sarawak (P1 & P2) 131
PerIis (P1 & P2) 141
8. Answer for CoIIection of SPM
TriaI 2008
155




















2
Ja0ap ge|j me:e-t, Jeat|aa, )ee-
FORMULAE FORMULAE FORMULAE FORMULAE

The following formulae may be helpful in answering the questions. The symbols given are the ones
commonly used.

ALGEBRA
1.
2
4
2
b b ac
x
a

=
2. a
m
a
n
= a
m + n

3. a
m
a
n
= a
m n

4. (a
m
)
n
= a
nm

5. log
a
mn = log
a
m + log
a
n
6. log
a

n
m
= log
a
m log
a
n
7. log
a
m
n
= n log
a
m
8.
a
b
b
c
c
a
log
log
log =
9. d n a T
n
) 1 ( + =
10. [ ] d n a
n
S
n
) 1 ( 2
2
+ =
11.
1
=
n
n
ar T
12. ) 1 ( ,
1
) 1 (
1
) 1 (

= r
r
r a
r
r a
Sn
n n

13. 1 ,
1
<

r
r
a
S

CALCULUS
1.
dx
du
v
dx
dv
u
dx
dy
uv y + = = ,
2.
2
,
v
dx
dv
u
dx
du
v
dx
dy
v
u
y

= =
3.
dx
du
du
dy
dx
dy
=
4. Area under a curve

=
b
a
ydx or

=
b
a
xdy
5. Volume generated

=
b
a
dx y
2
or

=
b
a
dy x
2


GEOMETRY
1. Distance ( ) ( )
2
2 1
2
2 1
y y x x + =
2. Midpoint

+ +
=
2
,
2
2 1 2 1
y y x x

3.
2 2
y x r + =
4.
2 2

y x
yj xi
r
+
+
=
5. A point dividing a segment of a line

+
+
+
+
=
n m
my ny
n m
mx nx
y x
2 1 2 1
, ) , (
6. Area of triangle
) ( ) (
3 1 2 3 1 2 1 3 3 2 2 1
y x y x y x y x y x y x A + + + + =










3
Program Cemerlang Bersama SMT Zon Johor Program Cemerlang Bersama SMT Zon Johor Program Cemerlang Bersama SMT Zon Johor Program Cemerlang Bersama SMT Zon Johor
STATISTICS
1.
N
x
x

=
2.

=
f
fx
x
3.
2
2 2
) (
x
N
x
N
x x
=

=


4.
2
2 2
) (
x
f
fx
f
x x f
=


5. C
f
F N
L M
m


+ =
2
1

6. 100
0
1
=
P
P
I
7.

=
w
I x
I
i i

8.
)! (
!
r n
n
P
r
n

=
9.
! )! (
!
r r n
n
C
r
n

=
10. ) ( ) ( ) ( ) ( B A P B P A P B A P + =
11. 1 , ) (
1
= + = =

q p q p C r X P
r r
r
n

12. Mean, np =
13. npq =
14.


=
x
z

TRIGONOMETRY
1. Arc length, r s =
2. Area of sector,
2
2
1
r A =
3. 1 cos sin
2 2
= + A A
4. A A
2 2
tan 1 sec + =
5. A A ec
2 2
cot 1 cos + =
6. A A A cos sin 2 2 sin =
7. A A A
2 2
sin cos 2 cos =
A
A
2
2
sin 2 1
1 cos 2
=
=

8.
A
A
A
2
tan 1
tan 2
2 tan

=
9. B A B A B A sin cos cos sin ) sin( =
10. B A B A B A sin sin cos cos ) cos( =
11.
B A
B A
B A
tan tan 1
tan tan
) tan(

=
12.
C
c
B
b
A
a
sin sin sin
= =
13. A bc c b a cos 2
2 2 2
+ =
14. Area of triangle C absin
2
1
=


















4
Ja0ap ge|j me:e-t, Jeat|aa, )ee-
ADDITIONAL MATHEMATICS ADDITIONAL MATHEMATICS ADDITIONAL MATHEMATICS ADDITIONAL MATHEMATICS SPM FORMAT SPM FORMAT SPM FORMAT SPM FORMAT

PAPER 1 (3472/1) PAPER 1 (3472/1) PAPER 1 (3472/1) PAPER 1 (3472/1)

No. No. No. No. Items Items Items Items Remarks Remarks Remarks Remarks
1. Item Type One section onIy
2. Number of Questions 25 questions ( (( (A AA Answer nswer nswer nswer A AA AII) II) II) II)
3. TotaI Marks 80 marks
4. Test Period 2 hours
5. Construct Division i) KnowIedge 20%
ii) AppIication of skiII 80%
6. Context Covered AII topics Iearned from Form 4 and 5
7. Equipments/ TooIs Scientific CaIcuIator
Four-figures mathematicaI tabIe
GeometricaI instruments


PAPER PAPER PAPER PAPER 2 22 2 (3472/ (3472/ (3472/ (3472/2 22 2) )) )

No. No. No. No. Items Items Items Items Remarks Remarks Remarks Remarks
1. Item Type Structured Subjective Structured Subjective Structured Subjective Structured Subjective
Section A Section A Section A Section A Section B Section B Section B Section B Section C Section C Section C Section C
2. Number of Questions 6 5 4
3. Question to be answered Answer AII Answer AII Answer AII Answer AII Choose 4 44 4 Choose 2 22 2
4. TotaI Marks 40 marks 40 marks 20 marks
5. Range of Marks/Question 5 - 8 marks 10 marks 10 marks
6. Question Number 1 - 6 7 - 11 12 - 15
7. Test Period 2 hours 30 minutes
8. Construct Division i) AppIication of skiII 60%
ii) ProbIem SoIving 40%
9. Context Covered AII topics Iearned from Form 4 and 5
10. DifficuIty LeveI Low Medium High
Ratio 4 3 3
11. Equipments/ TooIs Scientific CaIcuIator
Four-figures mathematicaI tabIe
GeometricaI instruments

5
Program Cemerlang Bersama SMT Zon Johor Program Cemerlang Bersama SMT Zon Johor Program Cemerlang Bersama SMT Zon Johor Program Cemerlang Bersama SMT Zon Johor
RECOMMENDED TIME ALLOCATION FOR EACH QUESTION RECOMMENDED TIME ALLOCATION FOR EACH QUESTION RECOMMENDED TIME ALLOCATION FOR EACH QUESTION RECOMMENDED TIME ALLOCATION FOR EACH QUESTION










PAPER 1 PAPER 1 PAPER 1 PAPER 1 (2 hours = 120 minutes) (2 hours = 120 minutes) (2 hours = 120 minutes) (2 hours = 120 minutes)
MARKS/QUESTION MARKS/QUESTION MARKS/QUESTION MARKS/QUESTION ALLOCATED TIME ALLOCATED TIME ALLOCATED TIME ALLOCATED TIME
2 marks 3 33 3 minutes 5
3 marks 4.5 4.5 4.5 4.5 minutes 10
4 marks 5 55 5 minutes 10
TotaI Time = 110 minutes
Extra Extra Extra Extra = 10 minutes = 10 minutes = 10 minutes = 10 minutes



PAPER 2 PAPER 2 PAPER 2 PAPER 2 (2 hours (2 hours (2 hours (2 hours 30 minutes 30 minutes 30 minutes 30 minutes = 1 = 1 = 1 = 15 55 50 minutes) 0 minutes) 0 minutes) 0 minutes)
ALLOCATED TIME ALLOCATED TIME ALLOCATED TIME ALLOCATED TIME
SECTION SECTION SECTION SECTION
Each Section Each Section Each Section Each Section Each Question Each Question Each Question Each Question
A AA A 1 hour 10 10 10 10 minutes 6
B BB B 1 hour 12 12 12 12 minutes 4
C CC C hour 12 12 12 12 minutes 2
TotaI Time = 132 minutes
Extra Extra Extra Extra = 1 = 1 = 1 = 18 88 8 minutes minutes minutes minutes


lt. 1c lt. 1c lt. 1c lt. 1c 1c 'o c,c| c,c| c,c| c,c| ;oc .vc









1 MARK 1.5 MINUTES

B
Ja0ap ge|j me:e-t, Jeat|aa, )ee-

QUICK NOTES
1. Shortcuts
2. Tips
3. Quick tests
7
Program Cemerlang Bersama SMT Zon Johor Program Cemerlang Bersama SMT Zon Johor Program Cemerlang Bersama SMT Zon Johor Program Cemerlang Bersama SMT Zon Johor
FORM 4 FORM 4 FORM 4 FORM 4




Z
QUADRATIC EQUATIONS
Real and distinct roots
Intersecting at two points
b
2
4ac
Real and equal roots
Touches/ Tangent
No real roots
Does not intersect
f(x) is always positives, f(x) > 0
f(x) is always negative, f(x) < 0
= 0
> 0
< 0
0
Two Real Roots
O
O
x
x
y
y
SOR = ( + ) =
a
b
,
POR = () =
a
c

x
2
(SOR)x + POR = 0

2
+
2
= ( + )
2
2
o' c.c.t l(c.to '; o' c.c.t l(c.to '; o' c.c.t l(c.to '; o' c.c.t l(c.to ';

1. Factorisation, (x + a)(x + b) = 0
2. Formula,
a
ac b b
x
2
4
2

=
3. Completing the Square,
ax
2
+ bx + c = 0 (a = 1)
c
b b
x +

+
2 2
2 2
= 0
Smile
Sad
c| 1ct c| 1ct c| 1ct c| 1ct
Given and are the roots of the quadratic equation
0 3 3 2
2
= x x . Find the equation having roots of 2
and 2 and state the equation in general form, ax
2
+ bx +
c = 0 such that a, b and c are integers.
STEPS SOLUTION
1. Previous
roots
+ =

=

2. New roots 2 + 2 =

(2)(2) =

3. New
Equation
x
2
(SOR)x + POR = 0




1
FUNCTIONS
a
b
c
d
e
f
One-to-One
a
b
c
d
e
f
Many-to-One
a
b
c
d
e
f
One-to-Many
a
b
c
d
e
f
Many-to-Many
a
b
d
e
f
Domain = Set X = {a, b}
Codomain = Set Y = {d, e, f}
Range = {d, e}
Objects = a, b
Image = d, e, f
X Y
x y z
f
g
gf
o(otc cto o(otc cto o(otc cto o(otc cto
1;(c o' [c'. 1;(c o' [c'. 1;(c o' [c'. 1;(c o' [c'.to to to to
lcc cto lcc cto lcc cto lcc cto
x y
f
f
1

ff
1
(x) = x
f
1
f(x) = x

B
Ja0ap ge|j me:e-t, Jeat|aa, )ee-


J
QUADRATIC FUNCTIONS
O
x
y
p
q
Max. Point
O
x
y
p
q
Min. Point
Axis of
symmetry
Form General Completing the
Square
f(x) =
Similar
Significance

|ct, o' .(, |ct, o' .(, |ct, o' .(, |ct, o' .(,
General Form: f(x) = ax
2
+ bx + c
Step Determined
1 Value of a : Max. or Min.
2 Value of b
2
4ac:
Value of b
2
4ac:
Find the roots
3 y-intercept, c :
4
Axes of symmetry:
a
b
x
2
=
y = (substitute x obtained into
given function)

Completing the Square:
Step Determined
1 Value of a : Max. or Min.
2 Express in the form of
f(x) = a(x p)
2
+ q
Find the roots
3 y-intercept, c : from general
form f(x) = ax
2
+ bx + c
4 Axes of symmetry: x = p
Max. or Min. value = q
Max or Min. point (p, q)
lc(c.'tc lc(c.'tc lc(c.'tc lc(c.'tc ,c .(,, ,c .(,, ,c .(,, ,c .(,,
x
-b
a
(x a)(x + b) > 0
x > a x < -b
x
c
d
(x c)(x d) < 0
c < x < a
.c ' c ot .c ' c ot .c ' c ot .c ' c ot
Completing the square
method
Using the Axis of
symmetry,
a
b
x
2
=

Differentiation method
aamp|e: 4 6 2 ) (
2
+ + = x x x f
4
) 2 ( 2
6
2
) 2 ( 2
6
2 ) (
2 2
+

+ = x x f

2
1
2
3
2
4
2
9
2
3
2
2
2

+ =
+

+ =
x
x

2
3
) 2 ( 2
6
= = x
Substitute in f(x):
2
1
4
2
3
6
2
3
2 ) (
2
=
+

= x f

6 4 ) ( ' + = x x f
At max./min. point, f (x) = 0
2
3
0 6 4
=
= +
x
x

Substitute in f(x):
2
1
4
2
3
6
2
3
2 ) (
2
=
+

= x f

Minimum point =


2
1
,
2
3
Minimum point =


2
1
,
2
3
Minimum point =


2
1
,
2
3


J-q t|: : Find the max./min. point of 1 4 ) (
2
+ = x x x f
9
Program Cemerlang Bersama SMT Zon Johor Program Cemerlang Bersama SMT Zon Johor Program Cemerlang Bersama SMT Zon Johor Program Cemerlang Bersama SMT Zon Johor






INDICES & LOGARITHMS
J|ad Cat Ceac|ade
Find the value of the following (using calculator):
c c c c
log
10
5 log
10
3 =

=
3
5
log
10


=
3 log
5 log
10
10

o'co o'co o'co o'co
b a log log
b
a
log "`\ "`\ "`\ "`\ b a log log
b
a
log
log


. lcc o' .ccto . lcc o' .ccto . lcc o' .ccto . lcc o' .ccto
eg.i. 12 3 3
1
= +
n n

Solution:
1. .
2. .
Solution:
.
` ` ` ` ,. o' '.c ,. o' '.c ,. o' '.c ,. o' '.c
eg.ii 6 3 5 3
2
= +
n n

. . ,..
= + b a
9 3
log log = + d c
8 2
log log = + y x
8 4
log log







2
1
9 log 3 log , 2 25 log , 3 8 log , 3 27 log , 2 4 log
2
1
9 9 5 2 3 2
= = = = = =
Any relation between log
a
y and log
y
a ?
4
SIMULTANEOUS EQUATIONS
\ct ccto \ct ccto \ct ccto \ct ccto
"(('.to "(('.to "(('.to "(('.to
l.('c 3y + 2x = 4 linear equation
y
2
2xy + x
2
= 0 non-linear equation
$ $$ $ $ $$ $0 00 0 0 00 00 00 0 0 00 08 88 8 8 88 8l ll l l ll ll ll l l ll ll ll l l ll l0 00 0 0 00 0l ll l l ll ll ll l l ll l0 00 0 0 00 0 N NN N N NN N0 00 0 0 00 0l ll l l ll l0 00 0 0 00 00 00 0 0 00 00 00 0 0 00 0I II I I II II II I I II II II I I II II II I I II I

1o
Ja0ap ge|j me:e-t, Jeat|aa, )ee-

^ o' io.t, l(c.to vt, ^ o' io.t, l(c.to vt, ^ o' io.t, l(c.to vt, ^ o' io.t, l(c.to vt,
t,c t,c t,c t,c .c 'o '.c .c 'o '.c .c 'o '.c .c 'o '.c
aamp|e : 3 log ) 1 2 ( log
2
2 2
+ = + x x
Make sure the bases are the same
Separate the logarithms and the
constant.
3 log ) 1 2 ( log
2
2 2
= + x x
Convert the coefficient to log
a
a = 1
2 log 3 log ) 1 2 ( log
2
2
2 2
= + x x
3
2
2
2 2
2 log log ) 1 2 ( log = + x x
Form a single log
3
2 2 2
2 log
) 1 2 (
log =
+
x
x

Ignore log. and solve it
2
1
,
4
1
0 ) 1 2 )( 1 4 (
0 1 2 8
8 1 2
2
) 1 2 (
2
2
3
2
= =
= +
=
= +
=
+
x or x
x x
x x
x x
x
x

1
2
3
4
5
' '' ' o' io.t, l(c.to vt, o' io.t, l(c.to vt, o' io.t, l(c.to vt, o' io.t, l(c.to vt,
t,c t,c t,c t,c c''cct 'o '.c c''cct 'o '.c c''cct 'o '.c c''cct 'o '.c
aamp|e : ) 2 ( log ) 12 2 ( log
3 9
+ = + x x
Change to the same log base (choose
smaller base)
) 2 ( log
3 log
) 12 2 ( log
) 2 ( log
9 log
) 12 2 ( log
3 2
3
3
3
3
3
+ =
+
+ =
+
x
x
x
x

Eliminate the fraction
) 2 ( log 2 ) 12 2 ( log
) 2 ( log
2
) 12 2 ( log
3 3
3
3
+ = +
+ =
+
x x
x
x

Coefficient to power
2
3 3
) 2 ( log ) 12 2 ( log + = + x x
Ignore log.
2
) 2 ( ) 12 2 ( + = + x x
Solve quadratic equation
4 , 2
0 ) 4 )( 2 (
0 8 2
0 12 4 2 4
4 4 12 2
) 2 )( 2 ( ) 12 2 (
2
2
2
= =
= +
= +
= + +
+ + = +
+ + = +
x x
x x
x x
x x x
x x x
x x x

1
2
3
4
5
J-q t|: : Solve the equation 36 log log
9 3
= r .
11
Program Cemerlang Bersama SMT Zon Johor Program Cemerlang Bersama SMT Zon Johor Program Cemerlang Bersama SMT Zon Johor Program Cemerlang Bersama SMT Zon Johor


COORDINATE GEOMETRY
Distance
( ) ( )
2
2 1
2
2 1
y y x x +
Midpoint

+ +
2
,
2
2 1 2 1
y y x x

Area of triangle/quadrilateral
) ( ) (
3 1 2 3 1 2 1 3 3 2 2 1
y x y x y x y x y x y x A + + + + =


A point B dividing a line AC in
the ratio m : n

+
+
+
+
=
n m
my ny
n m
mx nx
y x
C A C A
B B
, ) , (
Gradient
1 2
1 2
x x
y y
m

= or
ercept x
ercept y
m
int
int

=
Parallel
2 1
m m =
Perpendicular 1
2 1
= m m
Equation of straight line General form ay + bx + c = 0
Gradient form y = mx + c
Intercept form 1 = +
b
y
a
x

A moving point P moves such a way that n m PB PA : : =
2 2 2 2
) ( ) ( ) ( ) (
B B A A
y y x x m y y x x n =
If m = n, the locus is perpendicular bisector.
A moving P moves in such a way that = 90 APB
1 =

B
B
A
A
x x
y y
x x
y y
Perpendicular 1
2 1
= m m
Locus
A point P moves along the circumference of a circle with
centre M(x, y) and the circle passes through point A(x, y)
and B(x, y)
AM PM =

A B C

m n

B
A

Find the equation of the locus of the moving point P(x, y) for each of the following cases:
a) The distance of point P from the point Q(3, 4) is 2 units.
b) Point P is equidistant from the points S(5, 6) and T(1, 0).
c) The distances of point P from the points A(3, 1) and B(0, 2) are in the ratio 2 : 1.
Ans: a) 0 21 8 6
2 2
= + + y y x x , b) 0 15 3 2 = + y x , c) 0 2 6 2
2 2
= + + + + y x y x
ca|c0 Je:t:

12
Ja0ap ge|j me:e-t, Jeat|aa, )ee-


STATISTICS
l''ct o' ,.c c.t. l''ct o' ,.c c.t. l''ct o' ,.c c.t. l''ct o' ,.c c.t.
Measures of Central Tendency Measures of Dispersion
Change in the value if
every data is .. Mean Mode Median Range Q
3
Q
1

Std.
Dev.
Variance

2

Added by k + k + k + k
No change No change No change No change
Multiply by m m m m m m m m
2


Measurement Representative
Measures
of Central
Tendency
Mean
Mode
Median
M..,.
e
a, .
n,
Measures
of
Dispersion
Standard
Deviation
Variance
Interquartile
Range
Range
S.,
t...-.,
a..,.
D.
e
v..
i,.

a
:
q

t
e

-
e
m
e
m

e
-

ca|c0 Je:t:
The set of data x
1
, x
2
, x
3
, x
n
has the
following information:
Old New
Mean 26
Mode 28
Median 25
Range 10
Interquartile range 5
Standard Deviation 2
Variance 4

For the new set of data 2x
1
+ 3, 2x
2
+ 3,
2x
3
+ 3, 2x
n
+ 3, state the new one.
ltc(c.t'c .c ltc(c.t'c .c ltc(c.t'c .c ltc(c.t'c .c (Q
3
Q
1
)
***Build this table to avoid confusion.
Q
1
= N
4
1
Q
3
= N
4
3

L Lower boundry
F Cum. Freq. before the quartile class
f Freq. of the quartile class
N Total freq.
C Class size
Formula for median, C
f
F N
L m
m


+ =
2
1

Find the interquartile range for
below grouped data.
x f F
11 13 3
14 16 5
17 19 6
20 22 2
23 25 8
ca|c0 Je:t:
13
Program Cemerlang Bersama SMT Zon Johor Program Cemerlang Bersama SMT Zon Johor Program Cemerlang Bersama SMT Zon Johor Program Cemerlang Bersama SMT Zon Johor




9
DIFFERENTIATION
! l(c.to o' 1.ct/ `o.' ! l(c.to o' 1.ct/ `o.' ! l(c.to o' 1.ct/ `o.' ! l(c.to o' 1.ct/ `o.'
=
dx
dy
m
T
= gradient of tangent to the curve at x = a.
Gradient of normal, m
N
, 1 =
T N
m m perpendicular to the
tangent.
Equation of tangent, y = m
T
x + c
1

Equation of normal, y = m
N
x + c
2

tangent
normal
x
y
O
y = f(x)
x = a
m =
dx
dy
= 0
x
y
O
y = f(x)
x
1
x
2

x
1
minimum point
x
2
maximum point
. 1c/ t.to.;/ ,. ot ,./ , . 1c/ t.to.;/ ,. ot ,./ , . 1c/ t.to.;/ ,. ot ,./ , . 1c/ t.to.;/ ,. ot ,./ ,
=
dx
dy
m = 0
Find the value of x when 0 =
dx
dy
(commonly solving by linear,
quadratic or cubic equation).
0
2
2
>
dx
y d
.. 0
2
2
<
dx
y d
..
upp||cat|ea:
` [.tc o' ,.c ` [.tc o' ,.c ` [.tc o' ,.c ` [.tc o' ,.c
Chain rule:
dx
dy
dt
dx
dt
dy
=
eg.: If x changes at the rate of
3cms
-1
3 =
dt
dx

Decreases Value ..
^ .'' ,.c ' "((o.to ^ .'' ,.c ' "((o.to ^ .'' ,.c ' "((o.to ^ .'' ,.c ' "((o.to
x
dx
dy
y
Change of x x
If the value of x decreases: x ..
Approximate change in y
Approximate value of y = new value of y
y
old
+ y
8
CIRCULAR MEASURES
oc'.c oc'.c oc'.c oc'.c



=
180
. rad and

=
180
.

rad
Length of arc, s = r , where in radian.
Area of sector,
2
2
1
r A =
Length of chord AB, L =

2
sin 2

r
o Perimeter of segment = AB + r
Area of segment, ) sin (
2
1
2
= r A

s
r
segment
A
B
O
sector


=
=180 radian
1( 1( 1( 1( ^.. ... . ..
.. . , .. .. .. ...
C Ci ir rc cu ul la ar r M Me ea as su ur re es s c cI Io os se eI Iy y r re eI Iu ut te ed d t to o T Tr ri ig go on no om me et tr ry y

14
Ja0ap ge|j me:e-t, Jeat|aa, )ee-






11
INDEX NUMBERS
eg.:
The composite index for the year
1995 based on 1990 is 125. The
cost of the items increases by 40%
from the year 1995 to the year
2000. Find the composite index for
the year 2000 based on the year
1990.
c lcc. c lcc. c lcc. c lcc. I
100
0
1
=
P
P
I
P
1
specific year
P
0
base year
o(otc lcc. o(otc lcc. o(otc lcc. o(otc lcc. I

=
w
Iw
I
I price index
w weightage
Weightage can be in the form of:
i) Bar chart
ii) Pie chart
iii) Percentage
iv) Ratio
v) etc..
125 140
175
1995 1990 2000
100
125
1990
1995
=
I
I

125
1995
= I
100
140
1995
2000
=
I
I

100
1990
1995
1995
2000
1990
2000
=
I
I
I
I
I
I

140
2000
= I
175
100
100
125
100
140
1990
2000
=
=
I
I
The composite index for the year
2005 based on 2003 is 115.4. The
cost of the items increases by 55%
from the year 2005 to the year 2007.
Find the composite index for the
year 2007 based on the year 2003.




ca|c0 Je:t:
10
SOLUTION OF TRIANGLES
[c'c [c'c [c'c [c'c oc [c'c oc [c'c oc [c'c oc [c'c "c. o' 1.'c "c. o' 1.'c "c. o' 1.'c "c. o' 1.'c
C
c
B
b
A
a
sin sin sin
= =
A bc c b a cos 2
2 2 2
+ =
C ab A sin
2
1
=
Given
2 sides
1 included angle
3 sides
OR
2 sides
1 included angle
2 sides
1 included angle

a
c
b
B
C
A
a
c
C
B
C
A
If a < c and A is an acute non-included angle, the
ambiguous case will occur.

= + 180 ' ACB B AC
ACB B AC = 180 '
c c c c [c'c o' "'coc .c [c'c o' "'coc .c [c'c o' "'coc .c [c'c o' "'coc .c
15
Program Cemerlang Bersama SMT Zon Johor Program Cemerlang Bersama SMT Zon Johor Program Cemerlang Bersama SMT Zon Johor Program Cemerlang Bersama SMT Zon Johor
FORM 5 FORM 5 FORM 5 FORM 5





Z
LINEAR LAW
Non-linear equation Y = m X + c
x
b
x ay + =
= +
x h
x
p
y + =
= +
k a T + = +
2
1
= +
b
ax y =
= +
x
mp y

=
= +
k yx
n
=
= +

x
y
X
Y

If obtained values are decimal numbers
write in at least 2 decimal places

Y
X
c
m
0
Y = mX + c
1
progressions

"t,ct oco "t,ct oco "t,ct oco "t,ct oco coct oco coct oco coct oco coct oco
Common .. difference, d
1 2
T T d =
ratio, r
1 , 0 ,
1
2
= r r
T
T
r
The n
th
term d n a T
n
) 1 ( + =
1
=
n
n
ar T
3 consecutive terms a, b, c,
2 1
d d =
b c a b =
a, b, c,
2 1
r r =
b
c
a
b
=
r > 1 r < 1
1
) 1 (

=
r
r a
S
n
n

r
r a
S
n
n

=
1
) 1 (

The sum of the first n terms
[ ] d n a
n
S
n
) 1 ( 2
2
+ =
[ ] l a
n
S
n
+ =
2

a first term
l - last term
for 1 1 < < r then n
r
a
S

1

1
=
n n n
S S T
1
=
n n n
S S T If the formula for S
n
is given
eg.
7 8 8
S S T =
Sum of T
n
to T
k


1
=
n k k n
S S S
eg.: Find the sum of T
3
to T
8
Solution:
2 8 8 3
S S S =



1B
Ja0ap ge|j me:e-t, Jeat|aa, )ee-

J
INTEGRATION
1 ,
1
1

+
=
+

n
n
ax
dx ax
n
n

c
n a
n b ax
dx b ax
n
+
+
+ +
= +

) 1 (
1 ) (
) (
Given ) ( ) ( x f b ax
dx
d
n
= + , then

+ + = c b ax dx x f
n
) ( ) (
lcc'tc ltc.' lcc'tc ltc.' lcc'tc ltc.' lcc'tc ltc.'
Equation of the
curve, f(x)
Equation of the gradient of
tangent to the curve
INTEGRATION INTEGRATION INTEGRATION INTEGRATION

dx
dx
dy

DIFFERENTIATION DIFFERENTIATION DIFFERENTIATION DIFFERENTIATION
dx
dy

"c. "c. "c. "c.

+ + =
3
2
2
0
2
) 12 4 ( dx x dx x A
OR
+ =

2
0
2
dx x A A
triangle

3
0
2
y = x
2

y
x
y = 4x + 12

+ =
4
1
4
1
2
) 4 ( ) 4 ( dx x dx x x A
OR
=

4
1
2
) 4 ( dx x x A A
triangle
Negative: below x-axis
Area = Area J + Area K

+ =

2
1
1
0
) ( ) ( dx x f dx x f A
0 1
y = 4x x
2

y
x
4
y = x + 4
J
0
1
y
x
2
K
f(x) = x(x 1)(x 2)
0
y
x
x = y
2
y

=
1
0
2
) ( dx y y A
1
`o'cc `o'cc `o'cc `o'cc
( )

+ +
=
3
2
2
2
0
2
2
) 12 4 ( dx x
dx x V


3
0
2
y = x
2

y
x
y = 4x + 12

+
=
4
1
2
4
1
2 2
) 4 (
) 4 (
dx x
dx x x V


1
2
+ = y x

+ =
2
1
) 1 ( dy y V
0 1
y = 4x x
2

y
x
4
y = x + 4
0
1
y
x
2
0
y
x
x = y
2
1

=
1
1
2 2
) 1 ( dy y V
1
y = x
2
1
1
About x-axis About y-axis
Area, A

b
a
dx x f ) (

b
a
dy y f ) (
Volume, V

b
a
dx y
2

b
a
dy x
2


17
Program Cemerlang Bersama SMT Zon Johor Program Cemerlang Bersama SMT Zon Johor Program Cemerlang Bersama SMT Zon Johor Program Cemerlang Bersama SMT Zon Johor




TRIGONOMETRIC FUNCTIONS
y = 2sin 2x + 1
Negative (-ve)
Graph reflected about x-axis
Coefficient of x
Number of cycle of graph in 360
Coefficient of sine
Amplitude in y-axis
y-intercept
New position of x-axis
|ct, t,c .(, |ct, t,c .(, |ct, t,c .(, |ct, t,c .(,
y
x
0
y = 2sin 2x
|
90
|
180
|
270
|
360
2
2
Cycle 1 Cycle 2
Amplitude
y
x
0
y = 2sin 2x + 1
|
90
|
180
|
270
|
360
2
2
Cycle 1 Cycle 2
3
1
1
Reflected
4
VECTORS
If vector a and b are parallel, therefore ..
If points A, B and C are collinear, therefore : BC k AB = or AC m AB = (common point exist)
If a OA = and b OB = , therefore: AB = .
If T is the midpoint of AB , therefore: OT =
If m = 2i + 3j and n = i 4j, therefore:
i) m + n = ..
ii) |m + n| =
iii) Vector unit in the direction of m + n =
If A(1, 3) and B(2, 5), find the vector AB :
= OA = OB .. = AB .. = ..
If y x h CD + = 2 and y hk x CD 2 8 + = , find the values of h and of k. (using COMPARING METHOD)
.

1B
Ja0ap ge|j me:e-t, Jeat|aa, )ee-








PERMUTATIONS & COMBINATIONS
cct.to cct.to cct.to cct.to
Keywords
Arrange
Position
n
P
r

o'.to o'.to o'.to o'.to
Combine
Select
n
C
r

Code
Choose
Number of arrangement
for letters A, B & C.
1. ABC
2. ACB
3. BAC
4. BCA
5. CAB
6. CBA
Keywords
Number of combination
for letters A, B & C.
ABC
ACB
BAC
BCA
CAB
CBA
1 combination (as long as the
combination consist of letters
A, B & C)
6 arrangements (position of
A, B & C is important)
x
1
-1
y = -sin x
y 2
-2
y = 2sin x
y
|
360
Amplitude
x
1
y = sin 2x
y
|
360
|
180
-1
x
1
-1
y = sin 3x
y
|
360
x
2
0
y = sin x + 1
y
|
360
1
-1
x
1
-1
y = -sin x - 1
y
|
360
0
-2
x
1
-1
y =|sin x|
y
|
360
x
1
-1
y = -|sin x|
y
|
360
19
Program Cemerlang Bersama SMT Zon Johor Program Cemerlang Bersama SMT Zon Johor Program Cemerlang Bersama SMT Zon Johor Program Cemerlang Bersama SMT Zon Johor



8
PROBABILITY DISTRIBUTIONS
o.' \t'cto o.' \t'cto o.' \t'cto o.' \t'cto
P(X = r) =
n
C
r
p
r
q
n r

Where;
X = discrete random variable
(variable can be counted)
p = probability of success in a
single trial
q = probability of failure in a
single trial
r = 1, 2, 3, .., n
n = number of sample

Mean, np =
Variance, npq =
2

Standard Deviation npq =
Example: SPM 2003, Paper 2, Q10(a)
Senior citizen make up 20% of the population of a
settlement. If 7 people are randomly chosen from the
settlement, find the probability that at least two of
them are senior citizen.
Easier to take ;
1 P(0, 1) = P(2, 3, 4, 5, 6, 7)
No. of sample
PERCENTAGE (%) PROBABILITY (p) 1
20% p = 0.2
2
a
t

l
e
a
s
t

t
w
o





2
,

3
,

4
,

5
,

6
,

7

4
P(X 2) = 1 [P(X=0) + P(X=1)]
5
= 1 [
7
C
0
(0.2)
0
(0.8)
7
+
7
C
1
(0.2)
1
(0.8)
6
]
= 1 (0.2097 + 0.3670)
= 0.4233
p (0.2) q (0.8)
0
1
2
3
4
5
6
7
7
6
5
4
3
2
1
0
Make table 3
`o.' \t'cto `o.' \t'cto `o.' \t'cto `o.' \t'cto


=
x
z
Where;
z = standard score or z-score


PROBABILITY
Probability of event A,
) (
) (
) (
S n
A n
A P =
o Probability of even A OR event B = P(A) + P(B)
o Probability of even C AND event D = P(C) P(D)

Cases:
i) Probability of obtaining the same colours = P(Red AND Red) OR P(Blue AND Blue)
ii) Probability of winning at least once = P(W, W) OR P(W, L) OR P(L, W)
OR: Using the concept of compliment : 1 P(L, L)
OR Union, +
AND Intersection,

2o
Ja0ap ge|j me:e-t, Jeat|aa, )ee-

























9
MOTION ALONG A STRAIGHT LINE
1. List the given formula of displacement, s, velocity, v and acceleration, a.
eg. : t t t s 20 2
2 3
+ =

2
6 16 t t v + =
6 4 = t a
2. Recognize the following keywords:
(a) v
max
or v
min
a = 0
(b) s
max
or s
min
v = 0
(c) Initial velocity, v, or acceleration, a t = 0
(d) Instantaneous rest v = 0
(e) Distance traveled during : eg. 5
th
second s
5
s
4

(f) Distance traveled at first .. : eg. 7 seconds Find the values of below:
i) s
max/min
v = 0
ii) s
7
= ?
iii) s
0
= ?
Distance = 2s
max
s
7
s
0


t t t s 20 2
2 3
+ =

20 4 2
2
+ = t t v
4 4 + = t a

Differentiate v
dt
ds
= Integrate, s vdt =


Differentiate a
dt
dv
=
Integrate, v adt =


21
Program Cemerlang Bersama SMT Zon Johor Program Cemerlang Bersama SMT Zon Johor Program Cemerlang Bersama SMT Zon Johor Program Cemerlang Bersama SMT Zon Johor

SPM QUESTIONS SPM QUESTIONS SPM QUESTIONS SPM QUESTIONS
1 11 1 Summary Summary Summary Summary of Paper 1 (SPM 2003 of Paper 1 (SPM 2003 of Paper 1 (SPM 2003 of Paper 1 (SPM 2003 - -- - 2008) 2008) 2008) 2008)
2 22 2 Summary Summary Summary Summary of Paper 2 (SPM 2003 of Paper 2 (SPM 2003 of Paper 2 (SPM 2003 of Paper 2 (SPM 2003 - -- - 2008) 2008) 2008) 2008)
3 33 3 Questions (SPM 2003 Questions (SPM 2003 Questions (SPM 2003 Questions (SPM 2003 - -- - 2008) 2008) 2008) 2008)

22
Ja0ap ge|j me:e-t, Jeat|aa, )ee-
SUMMARY OF SUMMARY OF SUMMARY OF SUMMARY OF SPM QUESTIONS BASED ON THE TOPICS (SUBTOPICS/SKILLS) SPM QUESTIONS BASED ON THE TOPICS (SUBTOPICS/SKILLS) SPM QUESTIONS BASED ON THE TOPICS (SUBTOPICS/SKILLS) SPM QUESTIONS BASED ON THE TOPICS (SUBTOPICS/SKILLS)

2003 2003 2003 2003 2004 2004 2004 2004 2005 2005 2005 2005
PAPER 1 PAPER 1 PAPER 1 PAPER 1
1 11 1
FUNCTIONS (Relation-ordered pairs) FUNCTIONS (Relation-diagram) FUNCTIONS (Inverse + Composite)
2 22 2
FUNCTIONS (Inverse + Composite) FUNCTIONS (Inverse + compare) FUNCTIONS (Inverse)
3 33 3
QUAD. EQN. (Roots) FUNCTIONS (composite) FUNCTIONS (Inverse + Composite)
4 44 4
QUAD. EQN. (Type of roots) QUAD. EQN. (Form equation) QUAD. EQN. (Types of roots)
5 55 5
LOGARITHMS (Expressing) QUAD. EQN. (Type of roots) QUAD. EQN. (Roots)
6 66 6
INDICES (Solve by log.) QUAD. FUNC. (completing t square) QUAD. FUNCT. (Max/Min)
7 77 7
PROGRESSIONS (AP d) INDICES (Equaling the base) INDICES (Factorise)
8 88 8
PROGRESSIONS (GP r, S) LOGARITHMS (Expressing) LOGARITHMS (log. index)
9 99 9
COOR. GEO. (Division of line segmt.) PROGRESSIONS (GP r) LOGARITHMS (Expressing)
10 10 10 10
LINEAR LAW (Comparing) PROGRESSIONS (APTn+Tn+1+Tn+2) PROGRESSIONS (AP/GP)
11 11 11 11
COOR. GEO. (Perpendicular) PROGRESSIONS (AP Tn) PROGRESSIONS (APTn, Sn)
12 12 12 12
VECTORS (Cartesian plane) PROGRESSIONS (GP recurring no.) PROGRESSIONS (GP r, n)
13 13 13 13
VECTORS (Comparing) LINEAR LAW (Comparing) LINEAR LAW (Comparing)
14 14 14 14
VECTORS (Resultant vector) COOR. GEO. (Perpendicular) COOR. GEO. (Perpendicular)
15 15 15 15
DIFFERENTIATION (Max/Min) COOR. GEO. (Locus, m : n) VECTORS (Cartesian plane)
16 16 16 16
DIFFERENTIATION(Small change) VECTORS (Cartesian plane) VECTORS (Resultant vector)
17 17 17 17
INTEGRATION (Basic integration) VECTORS (Comparing) TRIGO. FUNCT. (Trigo. Equation)
18 18 18 18
INTEGRATION (Area) TRIGO. FUNCT. (Trigo. Equation) CIRCULAR MEAS. (s = r)
19 19 19 19
CIRCULAR MEAS. (s = r) CIRCULAR MEAS. (s = r) DIFFERENTIATION (2
nd
order)
20 20 20 20
TRIGO. FUNCT. (Complementary ) DIFFERENTIATION (Basic) DIFFERENTIATION(Rate change
21 21 21 21
TRIGO. FUNCT. (Trigo. Equation) DIFFERENTIATION(Rate change INTEGRATION (Basic )
22 22 22 22
PERMUTATION (Codes) INTEGRATION (Definite integrals ) COMBINATION
23 23 23 23
COMBINATION PERMUTATION (Arrangement) STATISTICS (manipulating formulae)
24 24 24 24
NORMAL DISTRIBUTION PROBABILITY PROBABILITY
25 25 25 25
BINOMIAL DISTRIBUTION NORMAL DISTRIBUTION NORMAL DISTRIBUTION











23
Program Cemerlang Bersama SMT Zon Johor Program Cemerlang Bersama SMT Zon Johor Program Cemerlang Bersama SMT Zon Johor Program Cemerlang Bersama SMT Zon Johor
SUMMARY OF SPM QUESTIONS BASED ON THE TOPICS (SUBTOPICS/SKILLS) SUMMARY OF SPM QUESTIONS BASED ON THE TOPICS (SUBTOPICS/SKILLS) SUMMARY OF SPM QUESTIONS BASED ON THE TOPICS (SUBTOPICS/SKILLS) SUMMARY OF SPM QUESTIONS BASED ON THE TOPICS (SUBTOPICS/SKILLS)

2006 2006 2006 2006 2007 2007 2007 2007 2008 2008 2008 2008
PAPER 1 PAPER 1 PAPER 1 PAPER 1
1 11 1
FUNCTIONS FUNCTIONS (Relation) FUNCTIONS (Absolute, range)
2 22 2
FUNCTIONS (Mapping) FUNCTIONS (Absolute value) FUNCTIONS (Inverse-Composite)
3 33 3
QUAD. EQN. (Type of roots) FUNCTIONS (Composite-compare) FUNCTIONS (Composite)
4 44 4
QUAD. FUNC. (Max/Min) QUAD. EQN. (Roots, Type of roots) QUAD. EQN. (Roots)
5 55 5
QUAD. FUNC. (Inequalities) QUAD. FUNC. (Inequalities) QUAD. FUNC. (Completing t square)
6 66 6
INDICES (Equaling base) QUAD. FUNC. (Completing t square) QUAD. FUNC. (Inequalities)
7 77 7
LOGARITHMS (Equaling base) LOGARITHMS (Change base) INDICES (Equaling base)
8 88 8
LOGARITHMS (Linear) INDICES (Equaling base) LOGARITHMS (Equaling base)
9 99 9
PROGRESSIONS(AP-Sim.Eqn.Tn,Sn) PROGRESSIONS(AP or GP) PROGRESSIONS (GP r)
10 10 10 10
PROGRESSIONS(GP-Sim.Eqn.,S) PROGRESSIONS(AP d) PROGRESSIONS(AP-Tn)
11 11 11 11
LINEAR LAW (Comparing) PROGRESSIONS(GP S) PROGRESSIONS(GP S)
12 12 12 12
COOR. GEO. (Perpendicular) LINEAR LAW (Comparing) LINEAR LAW (Reduce)
13 13 13 13
VECTORS (Cartesian plane) COOR. GEO. (Parallel) COOR. GEO. (SL intercept form,locus)
14 14 14 14
VECTORS (Parallel - compare) COOR. GEO. (Area of triangle) COOR. GEO. (Area of triangle)
15 15 15 15
TRIGO. FUNCT. (Trigo. Equation) VECTORS (Resultant vector) VECTORS (non-parallel, non-zero)
16 16 16 16
CIRCULAR MEAS. (s = r, Area) VECTORS (unit vector) VECTORS (Resultant vector)
17 17 17 17
DIFFERENTIATION (Gradient) TRIGO. FUNCT. (Trigo. Equation) TRIGO. FUNCT. (Complementary )
18 18 18 18
DIFFERENTIATION (Max/Min) CIRCULAR MEAS. (s = r, Area) CIRCULAR MEAS. (s = r, Area)
19 19 19 19
DIFFERENTIATION(Small change) DIFFERENTIATION (Gradient) DIFFERENTIATION(Small change)
20 20 20 20
INTEGRATION (Area) DIFFERENTIATION (Max/Min) DIFFERENTIATION(Normal eqn.)
21 21 21 21
INTEGRATION (Definite integrals) INTEGRATION (Definite integrals) INTEGRATION (Definite integrals)
22 22 22 22
PERMUTATION (Codes) STATISTICS (Manipulating formulae) STATISTICS (Mean, effect +)
23 23 23 23
PROBABILITY COMBINATION PERMUTATION
24 24 24 24
STATISTICS (Variance) PROBABILITY PROBABILITY
25 25 25 25
NORMAL DISTRIBUTION NORMAL DISTRIBUTION NORMAL DISTRIBUTION












24
Ja0ap ge|j me:e-t, Jeat|aa, )ee-
SUMMARY OF SPM SUMMARY OF SPM SUMMARY OF SPM SUMMARY OF SPM QUESTIONS BASED ON THE TOPICS (SUBTOPICS/SKILLS) QUESTIONS BASED ON THE TOPICS (SUBTOPICS/SKILLS) QUESTIONS BASED ON THE TOPICS (SUBTOPICS/SKILLS) QUESTIONS BASED ON THE TOPICS (SUBTOPICS/SKILLS)

2003 2003 2003 2003 2004 2004 2004 2004 2005 2005 2005 2005
PAPER 2 PAPER 2 PAPER 2 PAPER 2
1 11 1
SIMULTANEOUS EQUATIONS SIMULTANEOUS EQUATIONS SIMULTANEOUS EQUATIONS
2 22 2
QUAD. FUNC. (completing the square) COOR. GEO. (m1m2=-1, ratio) DIFF. (gradient func., straight line)
3 33 3
DIFFERENTIATION TRIGO. FUNCT.(sketch graphs cos2x) PROGRESSIONS (AP)
4 44 4
CIRCULAR MEASURES STATISTICS (x /o
2
, modified data) STATISTICS (calc. median, o)
5 55 5
STATISTICS (Modified data) INTEGRATION (gradient-eqn curve)
DIFFERENTIATION (max/min)
TRIGO. (Prove, sketch graphs)
6 66 6
VECTORS PROGRESSIONS (GP) VECTORS
7 77 7
LINEAR LAW (log) LINEAR LAW (log) LINEAR LAW (not log)
8 88 8
TRIGO. (Prove, sketch graphs) VECTORS INTEGRATION (area, volume)
9 99 9
DIFFERENTIATION (dV/dt)
INTEGRATION (volume)
CIRCULAR MEASURES COOR. GEO. (straight line, locus)
10 10 10 10
PROB. DISTR. (binomial, normal) DIFF. (gradient eqn. dy/dx)
INTEGRATIONS (area, volume)
CIRCULAR MEASURES
11 11 11 11
COOR. GEO. (locus, area ) PROB. DISTR. (binomial, normal) PROB. DISTR. (binomial, normal)
12 12 12 12
MOTION ALONG A SL INDEX NUMBERS (composite) SOLUTIONS OF (Ambiguous)
13 13 13 13
INDEX NUMBERS (composite) SOLUTIONS OF TRIANGLES INDEX NUMBERS (composite)
14 14 14 14
LINEAR PROGRAMMING LINEAR PROGRAMMING LINEAR PROGRAMMING
15 15 15 15
SOLUTIONS OF TRIANGLES MOTION ALONG A SL MOTION ALONG A SL

























25
Program Cemerlang Bersama SMT Zon Johor Program Cemerlang Bersama SMT Zon Johor Program Cemerlang Bersama SMT Zon Johor Program Cemerlang Bersama SMT Zon Johor
SUMMARY OF SPM QUESTIONS BASED ON THE TOPICS (SUBTOPICS/SKILLS) SUMMARY OF SPM QUESTIONS BASED ON THE TOPICS (SUBTOPICS/SKILLS) SUMMARY OF SPM QUESTIONS BASED ON THE TOPICS (SUBTOPICS/SKILLS) SUMMARY OF SPM QUESTIONS BASED ON THE TOPICS (SUBTOPICS/SKILLS)

2006 2006 2006 2006 2007 2007 2007 2007 2008 2008 2008 2008
P PP PAPER 2 APER 2 APER 2 APER 2
1 11 1
SIMULTANEOUS EQUATIONS SIMULTANEOUS EQUATIONS SIMULTANEOUS EQUATIONS
2 22 2
FUNCTIONS (inverse, composite) COOR. GEO. (Div. line, Perpendicular) QUAD. FUNC. (completing the square)
3 33 3
PROGRESSIONS (AP) TRIGO. FUNC. (sketch graphs) PROGRESSIONS (GP)
4 44 4
TRIGO. FUNC. (sketch graphs) DIFF. (Turning point, Integrate) TRIGO. FUNC. (Prove, sketch graphs)
5 55 5
VECTORS STATISTICS (interquartile range) STATISTICS (median)
6 66 6
STATISTICS (median, histgrm, mode) PROGRESSIONS (AP) VECTORS
7 77 7
LINEAR LAW (log) LINEAR LAW (not log) INTEGRATIONS (tangent., A, V)
8 88 8
INTEGRATIONS (sim. eqn., A, V) VECTORS LINEAR LAW (log)
9 99 9
COOR. GEO. (area, straight line, locus) CIRCULAR MEASURES CIRCULAR MEASURES
10 10 10 10
CIRCULAR MEASURES INTEGRATIONS (dy/dx., A, V) COOR. GEO. (locus, area)
11 11 11 11
PROB. DISTRIBUTION (normal) PROB. DIST. (Binomial, normal) PROB. DISTRIBUTION (normal)
12 12 12 12
MOTION ALONG A SL MOTION ALONG A SL MOTION ALONG A SL
13 13 13 13
SOLUTIONS OF TRIANGLES INDEX NUMBERS (composite) INDEX NUMBERS (composite)
14 14 14 14
LINEAR PROGRAMMING LINEAR PROGRAMMING SOLUTIONS OF (Ambiguous)
15 15 15 15
INDEX NUMBERS (composite) SOLUTIONS OF (Ambiguous) LINEAR PROGRAMMING



















2B
Ja0ap ge|j me:e-t, Jeat|aa, )ee-
PAPER 1 PAPER 1 PAPER 1 PAPER 1

FUNCTIONS


2003 2004 2005
R
E
L
A
T
I
O
N
S

1.

Based on the above information, the
relation between P and Q is defined
by the set of ordered pairs {(1,2),
(1,4), (2,6), (2,8)}.
State
(a) the image of 1
(b) the object of 2
[2 marks]


1.

Diagram above shows the relation
between set P and set Q.
State
(a) the range of the relation
(b) the type of the relation
[2 marks]

O
B
J
E
C
T
S

&

I
M
A
G
E
S




1.

In diagram above, the function h
maps x to y and the function g maps
y to z.
Determine
(a) h
-1
(5)
(b) gh(2)
[2 marks]
Ans: (a) 2, (b) 8
I
N
V
E
R
S
E

&

C
O
M
P
O
S
I
T
E

2.
Given that g : x 5x + 1 and
h: x x
2
2x + 3, find
(a) g
-1
(3)
(b) hg(x)
[4 marks]
Ans: (a)
5
2
, (b) 25x
2
+ 2
3.
Given the function 0 ,
6
) ( = x
x
x h
and the composite function
hg(x)=3x, find
(a) g(x)
(b) the value of x when gh(x)= 5
[4 marks]
Ans: (a)
x
2
, (b) x = 15
2.
The function w is defined as
2 ,
2
5
) (

= x
x
x w .
(a) w
-1
(x),
(b) w
-1
(4)
[3 marks]
Ans: (a) 0 ,
5 2

x
x
x
, (b)
4
3

3.
The following information refers to
the functions h and g.

Find gh
-1
(x)
[3 marks]
Ans: 2x + 5
C
O
M
P
A
R
I
N
G

2.
Given the function m x x h + 4 :
and
8
5
2 :
1
+

kx x h , where m and
k are constant, find value of m and
of k.
[3 marks]
Ans:
2
5
,
8
1
= = m k



h: x 2x 3
g: x 4x 1
8
2
5
x y z
h
g
d
e
f
w
x
y
z
Set P Set Q
P = {1,2,3}
Q = {2,4,6,8,10}
27
Program Cemerlang Bersama SMT Zon Johor Program Cemerlang Bersama SMT Zon Johor Program Cemerlang Bersama SMT Zon Johor Program Cemerlang Bersama SMT Zon Johor

2006 2007 2008
1.

In diagram above, set B shows the
images of certain elements of set A.
(a) State the type of relation between
A and B.
(b) Using the function notation, write
a relation between set A and set
B.
[2 marks]
Ans: (b) f : x x
2

1.

Diagram above shows the linear
function h
(a) State the value of m
(b) Using the function notation,
express h in terms of x.
[2 marks]
Ans: (a) m = 3, (b) h(x) = x + 1


2.

Diagram above shows the function
0 , :

x
x
x m
x h , where m is a
constant.
Find the value of m.
[2 marks]
Ans: m = 4
2.
Given 3 : x x f , find the value of
x such that f(x)= 5
[2 marks]
Ans: x = 8 or 2
1.

Diagram above shows the graph of
the function 1 2 ) ( = x x f , for the
domain 5 0 x .
State
(a) the value of t
(b) the range of f(x) corresponding to
the given domain.
[3 marks]
Ans: (a)
2
1
= t , (b) 0 f(x) 9
Paper 2, Q2
Given that f : x 3x 2 and
g: x 1
5
+
x
, find
(a) ) (
1
x f

,
[1 mark]
(b) ) (
1
x g f

,
[2 marks]
(c) h(x) such that hg(x) = 2x + 6
[3 marks]
Ans: a)
3
2 + x
, (b)
15
15 + x
,
(c) h(x )= 10x - 4
2.
Given the function 2 5 : + x x g
and 3 4 :
2
+ x x x h , find
(a) g
-1
(6)
(b) hg(x)
[4 marks]
Ans: (a)
5
4
, (b) 25x
2
1
3.
Given the functions f(x) = x 1 and
g(x) = kx + 2, find
(a) f(5)
(b) the value of k such that gf(5)=14
[3 marks]
Ans: (a) 5, (b) k = 3

3.
The following information is about
the function h and the composite
function h
2



Find the value of a and b.
[3 marks]
Ans: a = 6, b = 5




t
y = f(x)
x 5
1
0
y
h
x
x
x m

8
2
1

0
1
m
5
1
2
4
6
x h(x)
5
4
-4
-5

25
16
Set A Set B
h:x ax +b, where a and b are
constants and a > 0
h
2
:x 36x 35

2B
Ja0ap ge|j me:e-t, Jeat|aa, )ee-
QUADRATIC EQUATIONS/FUNTIONS


2003 2004 2005
F
i
n
d

R
o
o
t
s

3.
Solve the quadratic equation
2x(x 4) = (1 x)(x +2). Give your
answer correct to four significant
figures.
[ marks]
Ans: 2.592 or 0.2573
5.
Solve the quadratic equation
x(2x 5) = 2x 1. Give your
answer correct to three decimal
places.
[ marks]
Ans: 8.153 or 0.149
C
o
n
d
.

o
f

R
o
o
t
s


The quadratic equation
x(x + 1) = px 4 has two distinct
roots. Find the range of values of p.
[ marks]
Ans: p <3, p > 5
4.
The straight line of y = 5x 1 does
not intersect the curve
y = 2x
2
+ x + p. Find the range of
values of p.
[ marks]
Ans: p < 1


I
n
e
q
u
a
l
i
t
i
e
s

5.
Find the range of values of x for
which x(x 4) _ 12.
[3 marks]
Ans: 2 x 6


M
a
x
i
m
u
m
/

M
i
n
i
m
u
m

a
n
d

C
o
m
p
l
e
t
i
n
g

t
h
e

S
q
u
a
r
e
s

Paper 2, Q2
The function f(x)=x
2
4kx + 5k
2
+ 1
has a minimum value of r
2
+ 2k,
where r and k are constants.
(a) By using the method of
completing the square, show
that r = k 1.
[4 marks]
(b) Hence, or otherwise, find the
values of k and r if the graph of
the function is symmetrical
about 1
2
= r x .
[4 marks]
Ans: (b) r = 3, 1, k = 4, 0

6.

Diagram above shows the graph of
the function y = (x k)
2
2 where
k is a constant.
Find
(a) the value of k,
(b) the equation of the axis of
symmetry,
(c) the coordinates of the
maximum point.
[3 marks]
Ans: (a) k = 1, (b) x = 1, (c) (1, 2)

6.

Diagram above shows the graph of
a quadratic functions
f(x)= 3(x + p)
2
+ 2, where p is a
constant. The curve y = f(x) has the
minimum point (1, q), where q is a
constant. State
(a) the value of p,
(b) the value of q,
(c) the equation of the axis of
symmetry.
[3 marks]
Ans: (a) p = 1, (b) q = 2, (c) x = 1







G
i
v
e
n

R
o
o
t
s

4.
Form the quadratic equation which
has roots 3 and
2
1
. Give your
answer in the form
ax
2
+ bx + c = 0, where a, b and c
are constants.
[2 marks]
Ans: 2x
2
+ 5x 3 = 0




(2, 3)

3
0
y
x

(1, q)

y = f(x)
x
y
0
29
Program Cemerlang Bersama SMT Zon Johor Program Cemerlang Bersama SMT Zon Johor Program Cemerlang Bersama SMT Zon Johor Program Cemerlang Bersama SMT Zon Johor


2006 2007 2008
4(a)
Solve the following quadratic
equation
3x
2
+ 5x 2 = 0
Ans: 2 ,
3
1
= x


3.
A quadratic equation
x px x 2 9
2
= + + has two equal roots.
Find the possible values of p.
[3 marks]
Ans: p = 8, 4

4(b)
The quadratic equation
hx
2
+ kx + 3 = 0, where h and k are
constants, has two equal roots.
Express h in terms of k.
[4 marks]
Ans:
12
2
k
h =

5.
Find the range of the values of x for
x x x + > + 4 ) 4 )( 1 2 ( .
[2 marks]
Ans: x < 4, x > 1


5.
Find the range of values of x for
which x x + 1 2
2
.
[3 marks]
Ans: 1
2
1
x
6.
Find the range of values of x for
which x x < 5 ) 3 (
2
.
[3 marks]
Ans: 1 < x < 4
4.

Diagram above shows the graph of a
quadratic function y = f(x). The
straight line y = 4 is a tangent to the
curve y = f(x).
(a) Write the equation of the axis of
symmetry of the curve.
(b) Express f(x) in the form
(x + b)
2
+ c, where b and c are
constant
[3 marks]
Ans: (a) x = 3, (b) f(x)=(x 3)
2
- 4
6.
The quadratic function
4 2 ) (
2
+ = x x x f
can be expressed in the form
n m x x f + =
2
) ( ) ( , where m and n
are constants. Find the value of m and
n.
[3 marks]
Ans: m = 1, n = 5
5.
The quadratic function
f(x)=p(x + q)
2
+ r, where p, q and r are
constants, has a minimum value of
4. The equation of the axis of
symmetry is x = 3. State
(a) the range of values of p
(b) the value of q
(c) the value of r [3 marks]
Ans: (a) p > 0, (b) q = 3, (c) r = 4

Paper 2, Q2
Diagram below shows the curve of a
quadratic function f(x) = x
2
+ kx 5.
The curve has a maximum point at
B(2, p) and intersects the f(x)-axis at
point A.

(a) Find the coordinate of A.
[1 mark]
(b) By using the method of
completing the square, find the
value of k and of p [4 marks]
(c) Determine the range of values of
x, if f(x) _ 5. [2 marks]
Ans: (a) A(0, 5), (b) k = 4, p = 1,
(c) 4 0 x
4.
It is given that 1 is one of the roots
of the quadratic equation
x
2
4x p = 0.
Find the value of p.
[2 marks]
Ans: p = 5
B(2,p)

A
f(x)
x
0
y = 4
y = f(x)
1 5
x
y
0

3o
Ja0ap ge|j me:e-t, Jeat|aa, )ee-
PROGRESSIONS


2003 2004 2005
C
o
m
m
o
n

D
i
f
f
.

d
,

C
o
m
m
o
n

R
a
t
i
o
,

r

7(a)
The first three terms of an
arithmetic progression are k 3,
k + 3, 2k + 2. Find
(a) the value of k,
Ans: k = 7

9.
Given a geometric progression y, 2,
y
4
, p, , express p in terms of y.
[2 marks]
Ans:
2
8
y
p =
10.
The first three terms of a sequence
are 2, x, 8.Find the positive value of
x so that the sequence is
(a) an arithmetic progression
(b) a geometric progression
[2 marks]
Ans: (a) x =5;(b) x = 4
11(a)
The first three terms of an
arithmetic progression are 5, 9, 13.
Find
(a) the common difference of the
progression,
Ans: 4
T
E
R
M
S

8(a)
In a geometric progression, the first
term is 64 and the fourth term is 27.
Calculate
(a) the common ratio,
Ans:
4
3


11.
The volume of water in a tank is
450 litres on the first day.
Subsequently, 10 litres of water is
added to the tank everyday.
Calculate the volume, in litres, of
water in the tank at end of the 7
th

day.
[2 marks]
Ans: 510

S
u
m

o
f

T
e
r
m
s

7(b)
the sum of the first 9 terms of
progression.
[3 marks]
Ans: 252
10.
Given a arithmetic progression 7,
3, 1, ., state three consecutive
terms in this progression which sum
up to 75.
[3 marks]
Ans: 29, 25, 21
11(b)
(b) the sum of the first 20 terms
after the 3
rd
term.
[4 marks]
Ans: 1100

12.
The sum of the first n terms of the
geometric progression 8, 24, 72,
is 8744
Find
(a) the common ratio of the
progression
(b) the value of n
[4 marks]
Ans: (a) 3, (b) n = 7
S
o
l
v
e

b
y

S
i
m
u
l
t
a
n
e
o
u
s

E
q
u
a
t
i
o
n
s













S
u
m

t
o

I
n
f
i
n
i
t
y

8(b)
the sum to infinity of the geometric
progression.
[3 marks]
Ans: 256
12.
Express the recurring decimal
0.969696 as a fraction in its
simplest form.
[4 marks]
Ans:
33
32




31
Program Cemerlang Bersama SMT Zon Johor Program Cemerlang Bersama SMT Zon Johor Program Cemerlang Bersama SMT Zon Johor Program Cemerlang Bersama SMT Zon Johor


2006 2007 2008
10.
Three consecutive terms of an
arithmetic progression are
5 x, 8, 2x. Find the common
difference of the progression
[3 marks]
Ans:14
9.
(a) Determine whether the following
sequence is an arithmetic
progression or a geometric
progression.
16x, 8x, 4x
(b) Give a reason for the answer in
(a)
[2 marks]
Ans:(a) GP, (b) Ratio is constant
9.
It is given that the first four terms of a
geometric progression are 3, 6, 12
and x. find the value of x.
[2 marks]
Ans: x = 24
10.
The first three terms of arithmetic
progression are 46, 43 and 40. The n
th

term of this progression is negative.
Find the least value of n.
[3 marks]
Ans : n = 17












9.
The 9
th
term of an arithmetic
progression is 4 + 5p and the sum of
the first four terms of the progression
is 7p 10, where p is a constant.
Given that the common difference of
the progression is 5, find the value of
p.
[3 marks]
Ans: p = 8
10.
The third term of a geometric
progression is 16. The sum of the
third and the fourth term is 8.
Find
(a) the first term and common ratio
of the progression
Ans: a = 64

(b) the sum to infinity of the
progression.
[4 marks]
Ans:
3
2
42

11.
The first three terms of a geometric
progression are 27, 18, 12. Find the
sum to infinity of the geometric
progression.
[3 marks]
Ans: 81
11.
In geometric progression, the first
term is 4 and the common ratio is r.
Given that the sum to infinity of this
progression is 16, find the value of r.
[2 marks]
Ans: r =
4
3




32
Ja0ap ge|j me:e-t, Jeat|aa, )ee-
INDICES & LOGARITHMS

2003 2004 2005
E
q
u
a
l
i
n
g

t
h
e

b
a
s
e

7.
Solve the equation
32
4x
= 4
8x + 6

[3 marks]
Ans: x = 3


S
o
l
v
e

b
y

u
s
i
n
g

l
o
g
.

6.
Solve the equation 4
2x 1
= 7
x

[4 marks]
Ans: x = 1.677





F
a
c
t
o
r
i
s
e

7.
Solve the equation 2
x + 4
2
x + 3
= 1
[3 marks]
Ans: x = 3

E
q
u
a
l
i
n
g

b
a
s
e









l
o
g
.



i
n
d
e
x

8.
Solve the equation
log
3
4x log
3
(2x 1) = 1
[3 marks]
Ans:
2
3
= x

E
x
p
r
e
s
s
i
n
g

5.
Given that
log
2
T log
4
V = 3, express T in
terms of V.
[4 marks]
Ans: V T 8 =

8.
Given that log
5
2 = m, and
log
5
7 = p, express log
5
4.9 in terms
of m and p.
[4 marks]
Ans: 2p m 1
9.
Given that log
m
2 = p and
log
m
3 = r, Express

4
27
log
m
m
in
terms of p and r.
[4 marks]
Ans: 3r 2p + 1





















33
Program Cemerlang Bersama SMT Zon Johor Program Cemerlang Bersama SMT Zon Johor Program Cemerlang Bersama SMT Zon Johor Program Cemerlang Bersama SMT Zon Johor


2006 2007 2008
6.
Solve the equation
2
3 2
4
1
8
+

=
x
x
.
[3 marks]
Ans: x = 1
8.
Given the value of
n n
27 ) 3 ( 9
1
=

, find
the value of n.
[3 marks]
Ans: n = 0.5
Solve the equation 16
2x 3
= 8
4x

[3 marks]
Ans: x = 3
















8.
Given that 3 log log
2 4
= x , find the
value of x.
[3 marks]
Ans: x = 9

8.
Solve the equation
2 + log
3
(x 1) = log
3
x
[3 marks]
Ans:
8
1
1 = x


7.
Given that
log
2
xy = 2 + 3 log
2
x log
2
y, express
y in terms of x.
[3 marks]
Ans: y = 4x

7.
Given that x b =
2
log and y c =
2
log ,
express
c
b 8
log
4
in terms of x and y.
[4 marks]
Ans:
2 2 2
3 y x
x + =






















34
Ja0ap ge|j me:e-t, Jeat|aa, )ee-
LINEAR LAW

2003 2004 2005
S
u
b
s
t
i
t
u
t
e

U
n
k
n
o
w
n

I
n
t
o

R
e
d
u
c
e
d

E
q
n
.

13.

Diagram above shows a straight
graph of
x
y
against x. Given that
y = 6x x
2
, calculate the value of k
and of h.
[3 marks]
Ans: k = 4, h = 3

13.
The variables x and y are related by
the equation y = kx
4
, where k is a
constant,
(a) Convert the equation
y = kx
4
linear form.
(b) Diagram below shows the
straight line obtained by
plotting log
10
y against log
10
x.

Find
(i) log
10
k,
(ii) h.
[4 marks]
Ans: (b)(i) k = 1000, h = 11
R
e
d
u
c
e

&

C
o
m
p
a
r
e

10.

x and y are related by the equation y
= px
2
+ qx, where p and q are
constants. A straight is obtained by
plotting
x
y
against x. Calculate the
value of p and of q.
[4 marks]
Ans: p = 12, q = 13





























O
(0, 3)
(2, h)


log10
y
log10
x
O
(6,1)
(2, 9)


x
y

x
O
(h, 3)
(2, k)


x
y

x
35
Program Cemerlang Bersama SMT Zon Johor Program Cemerlang Bersama SMT Zon Johor Program Cemerlang Bersama SMT Zon Johor Program Cemerlang Bersama SMT Zon Johor


2006 2007 2008
12.

The variables x and y are related by
the equation ) 10 ( 2
2
x x y = . A
straight line graph is obtained by
plotting
x
y
2
against x as shown in the
diagram above. Find the value of p
and q.
[3 marks]
Ans: p = 10, q = 14





11.

Diagram (a) above shows the curve
5 3
2
+ = x y .
Diagram (b) above shows the straight
line graph obtained when
5 3
2
+ = x y is expressed in the linear
form Y = 5X + c.
Express X and Y in term of x and/or
y.
[3 marks]
Ans: y Y x X
5
3
,
25
9
2
= =

12.

The variables x and y are related by
the equation
x
k
y
5
= , where k is a
constant. Diagram above shows a
straight line graph is obtained by
plotting log
10
y against x.
(a) Express the equation
x
k
y
5
= in
its linear form to obtain the
straight line graph shown in
diagram above.
(b) Find the value k.
[4 marks]
Ans: (a) log y = ( log 5)x = log k
(b)
100
1
= k

















O
5 3
2
+ = x y

x
y
O
x
3
y
Diagram (b) Diagram (a)
O


log10 y
x
(0, 2)
O
(3, q)


x
y
2

x
(p, 0)

3B
Ja0ap ge|j me:e-t, Jeat|aa, )ee-
COORDINATE GEOMETRY

2003 2004 2005
D
i
v
i
s
i
o
n

o
f

L
i
n
e

S
e
g
m
e
n
t

9.
The point A(2h, h), B(p, t), C(2p,
3t) are on a straight line. B devide
AC internally in the ratio 2 : 3.
Express p in terms of t.
[3 marks]
Ans: p = 2t




I
n
t
e
r
c
e
p
t

F
o
r
m









P
e
r
p
e
n
d
i
c
u
l
a
r

11.
The equation of two straight line
are 1
3 5
= +
x y
and 5y = 3x + 24.
Determine whether the lines are
perpendicular to each other.
[3 marks]
Ans: Perpendicular
14.

Diagram above shows a straight
line PQ with the equation
1
3 2
= +
y x
. The point P lies on the
x-axis and the point Q lies on the
y-axis.
Find the equation of the straight
line perpendicular to PQ and
passing through the point Q.
[3 marks]
Ans: 3
3
2
+ = x y
14.
The following information refers to
the equations of two straight lines,
JK and RT, which are
perpendicular to each other.

Express p in terms of k.
[2 marks]
Ans:
k
p

=
2
1

S
i
m
u
l
t
a
n
e
o
u
s

E
q
u
a
t
i
o
n
s









L
o
c
u
s

15.
The point A is (1, 3) and the point
B is (4, 6). The point P moves such
that PA:PB = 2 : 3. Find the
equation of the locus of P.
[3 marks]
Ans:5x
2
+ 5y
2
+ 50x 6y 118= 0

A
r
e
a
















O
x
y
Q
P JK : y = px + k
RT : y = (k 2)x + p
where p and k are constants
37
Program Cemerlang Bersama SMT Zon Johor Program Cemerlang Bersama SMT Zon Johor Program Cemerlang Bersama SMT Zon Johor Program Cemerlang Bersama SMT Zon Johor


2006 2007 2008





13.
The straight line 1
6
= +
h
y x
has a y-
intercept of 2 and is parallel to the
straight line y + kx = 0. Determine the
value of h and of k.
[3 marks]
Ans:
3
1
, 2 = = k h
13(a)

Diagram above shows a straight line
passing through S(3, 0) and T(0, 4)
(a) Write down the equation of the
straight line ST in the form
1 = +
b
y
a
x
.












12.

Diagram above shows the straight
line AB which is perpendicular to the
straight line CB at the point B. The
equation of the straight line CB is
y = 2x 1. Find the coordinate at B.
[3 marks]
Ans:

3
17
,
3
10
B



13(b)
(b) A point (x, y) moves such that PS
= PT. Find the equation of the
locus of P.
[4 marks]
Ans: 8y 6x 7 = 0
14.
The vertices of a triangle are A(5, 2),
B(4, 6) and C(p, 2). Given that the
area of the triangle is 30 unit
2
, find
the values of p.
[3 marks]
Ans: p = 9 or 21
14.
The points (0, 3), (2, t) and (2, 1)
are the vertices of a triangle. Given
that the area of the triangle is 4 unit
2
,
find the values of t.
[3 marks]
Ans: t = 5 or 9







O
x
y
T (0, 4)
S (3, 0)
O
x
y
C
A(0, 4)
B

3B
Ja0ap ge|j me:e-t, Jeat|aa, )ee-
VECTORS

2003 2004 2005
C
o
m
p
a
r
i
n
g

13.
p = 2a + 3b
q = 4a b
r = ha + (h k)b, where h and k are
constants
Use the above information to find
the value of h and k when
r = 3p 2p
[3 marks]
Ans:h = 2, k = 13
17.
Given A(-2, 6), B(4, 2) and
C(m, p), find the value of m and of
p such that BC AB 2 + = 10i 12j
[4 marks]
Ans: m = 6, p = 2


V
e
c
t
o
r
s

i
n

C
a
r
t
e
s
i
a
n

P
l
a
n
e

&

U
n
i
t

V
e
c
t
o
r

12.

Diagram above shows two vectors,
OP and QO. Express
(a) OP in the form

y
x

(b) OQin the form xi + yj.
[2 marks]
Ans: (a)

3
5
, (b) 8i + 4j
16.
Given that O(0, 0), A(3, 4) and
B(2, 16), find in terms of the unit
vector i and j ,
(a) AB
(b) the unit vector in the direction
of AB .
[4 marks]
Ans: (a)

12
5
, (b)

12
5
13
1


15.

Diagram above shows vectors OA
drawn on Cartesian plane.
(a) Express OA in term of

y
x

(b) Find the unit vector in the
direction of OA .
[2 marks]
Ans: (a)

5
12
, (b)

5
12
13
1

R
e
s
u
l
t
a
n
t

V
e
c
t
o
r

14.

Diagram above shows a
parallelogram ABCD with BED as a
straight line. Given that AB = 6p,
AD= 4q and DE = 2EB, express,
in terms of p and q:
(a) BD ,
(b) EC
[4 marks]
Ans: (a) 6p + 4q, (b) q p
3
8
2 +
16.

Diagram above shows a
parallelogram OPQR, drawn on
Cartesan plane. It is given that
OP = 6i + 4j and
PQ = 4i + 5j. Find PR .
[3 marks]
Ans: 10i + j
N
o
n
-
p
a
r
a
l
l
e
l

V
e
c
t
o
r
s
















P
R
x
y
O
Q
E
B
C
D
A
P(5, 3)
Q(8, 4)
x
y
O
2 4 6 8 10 12 O
2
4
6
y
x
39
Program Cemerlang Bersama SMT Zon Johor Program Cemerlang Bersama SMT Zon Johor Program Cemerlang Bersama SMT Zon Johor Program Cemerlang Bersama SMT Zon Johor


2006 2007 2008
14.
The points P, Q and R are collinear. It
is given that PQ = 4a 2b and
QR = 3a + (1 + k)b, where k is a
constant.
Find
(a) the value of k
(b) the ratio of PQ : QR.
[4 marks]
Ans: (a)
2
5
= k , (b) PQ : QR = 4 : 3

13.
Diagram below shows two vectors,
OA and AB .

Express
(a) OA in terms of

y
x
,
(b) AB in terms of xi + bj.
[2 marks]
Ans: (a)

3
4
, (b) 4i 8j

16.
The following information refer to the
vectors a and b .

=
4
1
,
8
2
b a . Find
(a) the vector b a 2 ,
(b) the unit vector in the direction of
b a 2
[4 marks]
Ans: (a)

12
5
, (b)

12
5
13
1



15.

Diagram above shows a rectangle
OABC and the point D lies on the
straight line OB. It is given that
OD = 3DB. Express OD, in terms of
x and y.
[3 marks]
Ans: ) 9 5 (
4
3
x y +


Diagram above shows a triangle PQR.
The point T lies on QR such that
QT : TR = 3 : 1
Express in terms of a and b :
(a) QR
(b) PT
[4 marks]
Ans: (a) 4a 6b, (b) b a
2
3
3 +
15.
The vector a and b are non-zero and
non-parallel. It is given that
b k a h ) 5 ( ) 3 ( = + , where h and k
are constants. Find the value of
(a) h,
(b) k.
[2 marks]
Ans (a) h = 3, (b) k = 5





9x
C
O
5y
B
A

D
5
A(4, 3)
x
y
O
B
T
Q
P
6b
4a
R

4o
Ja0ap ge|j me:e-t, Jeat|aa, )ee-
TRIGONOMETRIC FUNCTIONS

2003 2004 2005
C
o
m
p
l
e
m
e
n
t
a
r
y

A
n
g
l
e

20.
Given that tan = t,
0
0
< 0 < 90
0
, express, in terms of t
(a) cot ,
(b) sin (90 ).
[3 marks]
Ans: (a)
t
1
, (b)
1
1
2
+ t


T
r
i
g
o
.

E
q
u
a
t
i
o
n
s

21.
Solve the equation
6sec
2
A - 13tanA = 0, 0
0
_ x _ 360
0
.
[4 marks]
Ans: 33.69, 213.69
or 56.31, 236.31

18.
Solve the equation
cos
2
x sin
2
x = sin x for
0
0
_ x _ 360
0
.
[4 marks]
Ans: x = 30, 150, 270
17.
Solve the equation
3cos

2x = 8sin x 5 for
0
0
_ x _ 360
0
.
[4 marks]
Ans: x = 41.81, 138.19






CIRCULAR MEASURE

2003 2004 2005
L
e
n
g
t
h

o
f

A
r
c
,

s

=

r


19.

Diagram above shows a sector
ROS with O. The length of the arch
RS is 7.24 cm and the perimeter of
the sector ROS is 25 cm. Find the
value of , in rad.
[3 marks]
Ans: = 0.8153 rad.
19.

Diagram above shows a circle
with centre O. Given that the
length of the major arch AB is
45.51 cm, find the length, in cm,
of the radius.
(Use = 3.142).
[3 marks]
Ans: r = 7.675 cm

18.

Diagram above shows a circle
with centre O. The length of minor
arch AB is 16 cm and the angle of
the major sector AOB is 290.
Using a = 3.142, find
(a) the value of , in radians,
(Give your answer correct to
four significant figures).
(b) the length, in cm, of the radius
of the circle.
[3 marks]
Ans: (a) 1.22 rad. (b) r = 13.09 cm
A
r
e
a





















O
S
R

A
B
O
A
B
0.354 rad
O
41
Program Cemerlang Bersama SMT Zon Johor Program Cemerlang Bersama SMT Zon Johor Program Cemerlang Bersama SMT Zon Johor Program Cemerlang Bersama SMT Zon Johor


2006 2007 2008
17.
Given that sin = p, where p is a
constant and 90 _ 0 _ 180,. Find, in
terms of p:
(a) cosec ,
(b) sin 2.
[3 marks]
Ans: (a)
p
1
, (b)
2
1 2 p p
15.
Solve the equation
15sin
2
x = sin x + 4sin 30 for
0
0
_ x _360
0

[4 marks]
Ans: x = 2335, 15625 or
x = 19928, 34032

17.
Solve the equation
cot x + 2cos x = 0 for 0
0
_ x _ 360
0
.
[4 marks]
Ans: x = 90, 210, 270, 330






2006 2007 2008















16.

Diagram above shows sector OAB
with centre O and sector AXY with
centre A.
Given that OB= 10 cm, AY= 4 cm,
XAY= 1.1 radians and the length of
arch AB= 7 cm, calculate
(a) the value of , in radian,
(b) the area, in cm
2
, of the shaded
region.
[4 marks]
Ans: (a) = 0.7 rad (b) 26.2 cm
2

18.

Diagram above shows a sector BOC
of a circle with centre O. It is given
that AD = 8 cm and BA = AO = OD =
DC = 5 cm. Find
(a) the length, in cm, of the arc BC,
(b) the area, in cm
2
, of shaded
region.
[4 marks]
Ans: (a) 18.5 cm, (b) 80.5 cm
2

18.

Given that P, Q, R are points such
that OP = PQ and OPR = 90, find
(a) QOR, in radians,
(b) the area, in cm
2
, of shaded region.
[4 marks]
Ans: (a) 1.0473 rad. (b) 30.714 cm
2






10 cm
R
O
P
Q
O
B
A
X
Y
D C
B
A
O
1.85rad

42
Ja0ap ge|j me:e-t, Jeat|aa, )ee-
DIFFERENTIATION

2003 2004 2005
B
a
s
i
c

D
i
f
f
e
r
e
n
t
i
a
t
i
o
n

28.
Differentiate 3x
2
(2x 5)
4
with
respect to x.
[3 marks]
Ans: 6x(6x 5)(2x 5)
3

19.
Given that h(x) =
( )
2
5 3
1
x
,
evaluate h(1).
[4 marks]
Ans:
8
27


T
a
n
g
e
n
t
/

N
o
r
m
a
l

E
q
n
.












M
a
x
.
/

M
i
n
.

15.
Given that y = 14x(5 x), calculate
(a) the value of x when y is a
maximum,
(b) the maximum value of y.
[3 marks]
Ans: (a)
2
5
, (b)
2
175


R
a
t
e

o
f

C
h
a
n
g
e

Paper 2, Q9(a)
Diagram below shows a conical
container of diameter 0.6 m and
height 0.5 m. Water is poured into
the container at a constant rate of
0.2 m
3
s
-1
.

Calculate the rate of change of the
height of the water level at the
instant when the height of the
water level is 0.4m.
[Use a = 3.142;
Volume of cone = h r
2
3
1
]
[4 marks]
Ans: 1.105
21.
Two variables, x and y, are related
by the equation
x
x y
2
3 + = .
Given that y increases at a constant
rate of 4 units per second, find the
rate of change of x when x = 2.
[3 marks]
Ans:
5
8


20.
The volume of water, V cm
2
, in a
container is given by
h h V 8
3
1
3
+ = , where h cm is the
height of water in the container.
Water is poured into container at
the rate of 10 cm
3
s
-1
.
Find the rate of change of the
height of water, in cm s
-1
, at the
instant when its height is 2 cm.
[3 marks]
Ans: 0.8333 cms
-1

S
m
a
l
l

C
h
a
n
g
e
s

16.
Given that y = x
2
+ 5x, use
differentiation to find the small
change in y when x increases from
3 to 3.01.
[3 marks]
Ans: 0.11







Water
0
.
5

m

0.6
43
Program Cemerlang Bersama SMT Zon Johor Program Cemerlang Bersama SMT Zon Johor Program Cemerlang Bersama SMT Zon Johor Program Cemerlang Bersama SMT Zon Johor


2006 2007 2008
18.
It is given that
7
3
2
u y = , where
u = 3x 5.
Find
dx
dy
in terms of x.
[4 marks]
Ans: 14(3x 5)
6


17.
The point P lies on the curve
y = (x 5)
2
. It is given that the
gradient of the normal at P is
4
1
.
Find the coordinate of P.
[3 marks]
Ans: P(7, 4)
19.
The curve y = f(x) is such that
5 3 + = kx
dx
dy
, where k is a constant.
The gradient of the curve at x =2 is 9.
Find the value of k.
[2 marks]
Ans:
3
2
= k
20.
The normal to the curve x x y 5
2
=
at point P is parallel to the straight
line 12 + = x y . Find the equation of
the normal to the curve at point P.
[4 marks]
Ans: y = x 3
20.
The curve y = x
2
32x + 64 has a
minimum point at x = p, where p is a
constant. Find the value of p.
[3 marks]
Ans: p = 16


























19.
Given that 4 3
2
+ = x x y ,
(a) find the value of
dx
dy
when x = 1,
(b) express the approximate change
in y, in terms of p, when x
changes from 1 to 1 + p, where p
is a small value.
[4 marks]
Ans (a) 7, (b) 7p

19.
Two variables x and y, are related by
the equation
2
16
x
y = . Express, in
terms of h, the approximate change in
y, when x changes from 4 to 4 + h,
where h is a small value.
[3 marks]
Ans: 0.5h


44
Ja0ap ge|j me:e-t, Jeat|aa, )ee-
INTEGRATION

2003 2004 2005
B
a
s
i
c

I
n
t
e
g
r
a
t
i
o
n

17.
Given that
( )
c x k dx
x
n
+ + =
+

) 1 (
1
5
4
, find
the value of k and n.
[3 marks]
Ans: 3 ,
3
5
= = n k


22.
Given that

=
k
dx x
1
6 ) 3 2 ( ,
where 1 > k , find the value of k.
[4 marks]
Ans: k = 5


D
e
f
i
n
i
t
e

I
n
t
e
g
r
a
l
s

21.
Given that 7 ) (
6
2
=

dx x f and
[ ] 10 ) ( 2
6
2
=

dx kx x f , find the
value of k.
[4 marks]
Ans:
4
1
= k
A
r
e
a

18.

Diagram above shows the curve
y = 3x
2
and the straight line x = k. If
the area of shaded region is 64
unit
2
, find the value of k.
[3 marks]
Ans: k = 4





STATISTICS

2003 2004 2005
M
a
n
i
p
u
l
a
t
i
n
g

F
o
r
m
u
l
a
e


Paper 2, Q5
A set of examination marks x
1
, x
2
,
x
3
, x
4
, x
5
, x
6
has a mean of 5 and a
standard deviation of 1.5.
(a) Find
(i) the sum of the marks, _x,
(ii) the sum of the squares of the
marks, _x
2
.
[3 marks]
Ans: (a)(i) 30, (ii) 163.5
(b) Each mark is multiply by 2 and
then 3 is added to it. Find, for the
new set of marks,
(i) the mean,
(ii) the variance.
[4 marks]
Ans: (b)(i) 13, (ii) 9
Paper 2, Q4
A set of data consist of 10 numbers
is 150 and the sum of the squares of
the numbers is 2,472.
a) Find the mean and variance of
the 10 numbers.
[3 marks]
Ans: (a) 15, 22.2
b) Another number is added to the
set of data and the mean is
increase by 1. Find
i) the value of this missing
number,
ii) the standard deviation of
the set of 11 numbers.
[4 marks]
Ans: (b)(i) k = 26, (ii) 5.494
23.
The mean of four numbers is m .
The sum of the squares of the
numbers is 100 and the standard
deviation is 3k. Express m in terms
of k.
[3 marks]
Ans: m = 25 9k
2




y =
2
y
x
x = k
O
45
Program Cemerlang Bersama SMT Zon Johor Program Cemerlang Bersama SMT Zon Johor Program Cemerlang Bersama SMT Zon Johor Program Cemerlang Bersama SMT Zon Johor


2006 2007 2008
21.
Given

+ + = + c x px dx x
3 2
) 1 6 ( ,
where p and c are constants, find
(a) the value of p,
(b) the value of c if

= + 13 ) 1 6 (
2
dx x when x = 1
[3 marks]
Ans: p = 2, c = 10
21.
Given that

=
5
1
8 ) ( dx x g , find
a) the value of

1
5
) ( dx x g ,
b) the value of k if
[ ] 10 ) (
1
5
=

dx x g kx
[4 marks]
Ans:
2
3
= k

21.
Given that 3 ) (
7
2
=

dx x h , find
(a)

2
7
) ( dx x h
(b) [ ]


7
2
) ( 5 dx x h .
[4 marks]
Ans: (a) 3, (b) 22

20.

Diagram above shows the curve
y = f(x) cutting the x-axis at x = a and
x = b.
Given that the area of the shaded
region is 5 unit
2
, find of dx x f
b
a

) ( 2 .
[2 marks]
Ans: 10






2006 2007 2008
24.
A set of positive integers consist of 2,
5 and m. The variance for this set of
integers is 14. Find the value of m.
[3 marks]
Ans: m = 11
22.
A set of data consists of five numbers.
The sum of the numbers is 60 and the
sum of the squares of the numbers is
800. Find, for the five numbers
(a) the mean,
(b) the standard deviation.
[3 marks]
Ans: (a) 12, (b) 4








22.
A set of seven numbers has a mean of
9.
(a) Find _x.
(b) When a number k is added to this
set, the new mean is 8.5. Find the
value of k.
[3 marks]
Ans: (a) 63, (b) k = 5


i
a
i
b
x
y
O
y = f(x)

4B
Ja0ap ge|j me:e-t, Jeat|aa, )ee-
PERMUTATION AND COMBINATION

2003 2004 2005
P
e
r
m
u
t
a
t
i
o
n
s

22.

Diagram above shows five letters
and three digits.
A code is to be formed using those
letters and digits. The code must
consist of 3 letters and followed by
2 digits. How many codes can be
formed if no letters is repeated in
each code?
[3 marks]
Ans: 360
23.

Diagram above shows five cards of
difference letters.
(a) Find the number of possible
arrangements, in a row, of all
the cards.
(b) Find the number of these
arrangement in which the letter
E and A are side by side.
[4 marks]
Ans: (a) 120, (b) 48

C
o
m
b
i
n
a
t
i
o
n
s

23.
A badminton team consists of 7
students. The team will be chosen
from a group of 8 boys and 5 girls.
Find the number of teams that can
be formed such that each team
consist of
a) 4 boys,
b) not more than 2 girls.
[4 marks]
Ans: (a) 700, (b) 708
22.
A debating team consists of 5
students. These 5 students are
chosen from 4 monitors, 2 assistant
monitors and 6 prefects. Calculate
the number of different ways the
team can be formed if
a) no restriction,
b) the team contains only 1
monitor and exactly 3 prefects
[4 marks]
Ans: (a) 792, (b) 160





PROBABILITY


2003 2004 2005
B
a
s
i
c

P
r
o
b
a
b
i
l
i
t
y

24.
A box contains 6 white marbles and
k black marbles. If a marble is
picked randomly from the box,
probability of getting a black marble
is
5
3
. Find the value of k.
[3 marks]
Ans: k = 9

24.
Colour Number of
Cards
Black 5
Blue 4
Yellow 3

The table above shows the number
of coloured cards in a box.
Two cards are drawn at random
from the box.
Find the probability that both cards
are of the same colour.
[3 marks]
Ans:
66
19











A

B C D E 7 6 8
H E B A T
47
Program Cemerlang Bersama SMT Zon Johor Program Cemerlang Bersama SMT Zon Johor Program Cemerlang Bersama SMT Zon Johor Program Cemerlang Bersama SMT Zon Johor


2006 2007 2008
22.
Diagram shows seven letters cards

A four-letter code is to be formed
using four of these cards. Find
a) the number of different four-letter
codes that can be form,
b) the number of different four-letter
codes which end with consonant.
[4 marks]
Ans: (a) 840, (b) 480
23(b)
(b) the number of ways the team
can be arranged in a row for a
group photograph, if the three
girls sit next to each other.
[4 marks]
Ans: 36
23.
Diagram shows six numbered cards

A fourdigit numbers is to be formed
using four of these cards. How many
a) different number can be formed?
b) different odd numbers can be
formed?
[4 marks]
Ans: (a) 360, (b) 240
23(a)
A coach wants to choose 5 players
consisting 2 boys and 3 girls to form
badminton team. These 5 players are
chosen from a group of 4 boys and 5
girls.
(a) the number of ways the team can
be formed.
Ans: 60












2006 2007 2008
23.
The probability that Hamid qualifies
for the final of track event is
5
2
while
the probability that Mohan qualifies is
3
1
.
Find the probability that
(a) both of them qualify for the final,
(b) only one of them qualifies for the
final.
[3 marks]
Ans: (a)
15
2
, (b)
15
7



24.
The probability that each shot fired by
Ramli hits a target is
3
1
.
(a) If Ramli fires 10 shots, find the
probability that exactly 2 shots
hit the target.
(b) If Ramli fires n shots, the
probability that all the n shots hit
the target is
243
1
. Find the value
of n.
[4 marks]
Ans: (a) 0.1951, (b) n = 5
24.
The probability of Sarah being chosen
as a school prefect is
5
3
while the
probability of Aini being chosen is
12
7
.
Find the probability that
(a) neither of them is chosen as a
school prefect,
(b) only one of them is chosen as a
school prefect.
[4 marks]
Ans: (a)
6
1
, (b)
60
29












3 5 6 7 8 9 U N I F O R M

4B
Ja0ap ge|j me:e-t, Jeat|aa, )ee-
PROBABILITY DISTRIBUTION

2003 2004 2005
B
i
n
o
m
i
a
l

D
i
s
.

25.
In an examination, 70% of the students
passed. If a sample of 8 students is
randomly selected, find the probability
that 6 students from the sample passed
the examination.
[3 marks]
Ans: 0.2965



N
o
r
m
a
l

D
i
s
t
r
i
b
u
t
i
o
n
s

24.

Diagram above shows a standard
normal distribution graph.
If P(0 < z < k) = 0.3128, find P(z > k).
[2 marks]
Ans: 0.1872








25.
X is a random variable of a normal
distribution with a mean of 5.2
and a variance of 1.44. Find
a) the z-score if X = 6.7,
b) P(5.2 _ X _ 6.7)
[4 marks]
Ans: (a) z = 1.25, (b) 0.3944
25.
The mass of students in a school
has a normal distribution with
mean of 54 kg and a standard
deviation of 12 kg. Find
a) the mass of the students
which gives a standard score
of 0.5,
b) the percentage of students
with masses greater than 48
kg.
[4 marks]
Ans: (a) X = 60, (b) 69.146%








































O
z
f(z)
k
49
Program Cemerlang Bersama SMT Zon Johor Program Cemerlang Bersama SMT Zon Johor Program Cemerlang Bersama SMT Zon Johor Program Cemerlang Bersama SMT Zon Johor


2006 2007 2008










25.

Diagram above shows a standard
normal distribution graph.
The probability represented by the
area of the shaded region is 0.3485.
a) Find the value of k.
b) X is a continuous random
variable which is normally
distributed with a mean of 79 and
a standard deviation of 3. Find
the value of X when the z-score
is k.
[4 marks]
Ans: (a) k = 1.03, (b) X = 82.09
25.
X is a continuous random variable of
a normal distribution with a mean of
52 and a standard deviation of 10.
Find
a) the z-score when X = 67.2,
b) the value of k when P(z < k) =
0.8849
[4 marks]
Ans: (a) z = 1.52, (b) k = 1.2
25.
The masses of a group of students in a
school have a normal distribution
with mean of 40 kg and a standard
deviation of 5 kg. Calculate the
probability that a student chosen at
random from this group has a mass of
a) more than 45 kg
b) between 35 kg and 47.8 kg.
[4 marks]
Ans: (a) 0.1587, (b) 0.5536

































O
z
f(z)
k
0.3485

5o
Ja0ap ge|j me:e-t, Jeat|aa, )ee-
COMMONLY QUESTIONED COMMONLY QUESTIONED COMMONLY QUESTIONED COMMONLY QUESTIONED TOPICS TOPICS TOPICS TOPICS IN PAPER 2 IN PAPER 2 IN PAPER 2 IN PAPER 2

SECTION A & B SECTION A & B SECTION A & B SECTION A & B


2003 2004 2005
S
I
M
U
L
T
A
N
E
O
U
S

E
Q
U
A
T
I
O
N
S

1.
Solve the simultaneous equation
4x + y = 8 and x
2
+ x y = 2
[5 marks]
Ans: x = 2, 3, y = 0, 4









1.
Solve the simultaneous equation
p m = 2 and p
2
+ 2m = 8.
Give your answers correct to three
decimal places.
[5 marks]
Ans: m = 0.606, 6.606,
p = 2.606, 4.606
1.
Solve the simultaneous equation
1
2
1
= + y x and y
2
10 = 2x
[5 marks]
Ans: x =
2
1
, 3 , y = 4, 3
P
R
O
G
R
E
S
S
I
O
N
S

6.
The diagram shows the arrangement
of the first three of an infinite series
of similar triangles. The first triangle
has a base x cm and a height y cm.
The measurement of base and height
of each subsequent triangle are half
of the measurements of its previous
one.

(a) Show that the areas of triangles
form a geometric progression
and state the common ratio.
[3 marks]
(b) Given that x = 80 cm and y = 40
cm,
(i) determine which triangle
has an area of
4
1
6 cm
2
.
(ii) find the sum to infinity of
the areas, in cm
2
, of the
triangle.
[5 marks]
Ans: (a)
4
1
= r ,
(b)(i) 5
th
=
4
1
6 , (ii)
3
1
2133
3.
Diagram shows part of arrangement
of bricks of equal size.

The number of bricks in the lowest
row is 100. For each of the other
rows, the number of bricks is 2 less
than in the row below. The height of
each brick is 6 cm. Ali builds a wall
by arranging bricks in this way. The
number of bricks in the highest row
is 4. Calculate,
(a) the height, in cm, of the wall,
[3 marks]
(b) the total price of the bricks
used if the price of one brick is
40 sen.
[3 marks]
Ans: (a) 294 cm, (b) RM1,019.20












y
x
6 cm
51
Program Cemerlang Bersama SMT Zon Johor Program Cemerlang Bersama SMT Zon Johor Program Cemerlang Bersama SMT Zon Johor Program Cemerlang Bersama SMT Zon Johor





2006 2007 2008
1.
Solve the simultaneous equation
2x + y = 1 and 2x
2
+ y
2
+ xy = 5.
Give your answers correct to three
decimal places.
[5 marks]
Ans:







1.
Solve the following simultaneous
equation:
2x y 3 = 0 and
2x
2
10x + y + 9 = 0
[5 marks]
Ans: x = 3, 1, y = 3, 1
1.
Solve the following simultaneous
equation:
x 3y + 4 = 0
x
2
+ xy 40 = 0
[5 marks]
Ans: x = 6, 5, y = 3 ,
3
2

3.
Two companies, Delta and Omega,
start to sell cars at the same time.
(a) Delta sells k cars in the first
month, and its sales increase
constantly by m cars every
subsequent month. It sells 240
cars in the 8th month and the
total sales for the first 10 months
are 1900 cars.
Find the value of k and m.
[5 marks]
(b) Omega sells 80 cars in the first
month and its sales increase
constantly by 22 cars every
subsequent month.
If both companies sell the same
number of cars in the nth month,
find the value of n.
[2 marks]
Ans: (a) k = 100, m = 20 (b) n = 11
6.
Diagram below shows the side
elevation of part of stairs built of
cement blocks.



The thickness of each block is 15 cm.
The length of each subsequent block
is 30 cm less than the preceding
block.
(a) If the height of the stairs to be
built is 3 m, calculate
i) the length of the top most
block,
ii) the total length of the block.
[5 marks]
(b) Calculate the maximum height of
the stairs.
[3 marks]
Ans: (a)(i) 415 cm, (ii) 14000 cm
(b) 495




3.
Muthu started working for a company
on 1 January 2002 with an initial
annual salary of RM18000. Every
January, the company increased his
salary by 5% of the previous years
salary. Calculate
(a) his annual salary, to the nearest
RM, for the year 2007,
[3 marks]
(b) the minimum value of n such that
his annual salary in the n
th
year
will exceed RM36 000,
[2 marks]
(c) the total salary, to the nearest
RM, paid to him by the
company, for the years 2002 to
2007.
[2 marks]
Ans: (a) RM22,973, (b) n = 16,
(c) RM122,434












985 cm
925 cm
955 cm
15 cm

52
Ja0ap ge|j me:e-t, Jeat|aa, )ee-

2003 2004 2005
C
O
O
R
D
I
N
A
T
E

G
E
O
M
E
T
R
Y

2.
Diagram below shows a straight line
CD which meets a straight line AB
at the point D. The point C lies on
the y-axis.

(a) Write down the equation of AB
in the form of intercepts.
[1 marks]
(b) Given that 2AD = DB, find the
coordinate of D.
[2 marks]
(c) Given that CD is perpendicular
to AB, find the y-intercept of
CD.
[3 marks]
Ans: (a) 1
6 9
=
y x
, (b) D(3, 4),
(c)
2
1



9.
Solution of this question by scale
drawing will not be accepted.


(a) Find
i) the equation of the straight
line AB,
ii) the coordinate of B.
[5 marks]
(b) The straight line AB is
extended to a point D such that
AB : BD = 2 : 3. Find the
coordinate of D.
[2 marks]
(c) A point P moves such that its
distance from point A is always
5 units. Find the equation of the
locus of P.
[3 marks]
Ans: (a)(i) y = 2x + 17, (ii) B(8, 1),
(b) D(14, 11),
(c) x
2
+y
2
+8x18y+72=0
V
E
C
T
O
R
S

12.
Give that

=
3
2
,
7
5
OB AB and

=
5
k
CD , find
(a) the coordinate of A,
[2 marks]
(b) the unit vector in the direction
of OA ,
[2 marks]
(c) the value of k, if CD is parallel
to AB
[2 marks]
Ans: (a) (3, 4), (b)

4
3
5
1
,
(c)
7
25
= c
8.
Diagram below shows triangle
OAB. The straight line AP intersects
the straight line OQ at R. It is given
that OP =
3
1
OB, AQ =
4
1
AB,
x OP 6 = and y OA 2 = .

(a) Express in terms of x and/or y:
(i) AP
(ii) OQ [4 marks]
(b) (i) Given that AP h AR = , state
AR in terms of h, x and y.
(ii) Given that kOQ RQ = , state
RQ in terms of k, x and y.
[2 marks]
(c) Using AR and RQ from (b), find
the value of h and of k.
[4 marks]
Ans: (a)(i) 2y + 6x, (ii) x y
2
9
2
3
+
(b)(i) h(2y + 6x) (ii) k

+ x y
2
9
2
3
,
(c)
2
1
,
3
1
= = h k
6.
In diagram below, ABCD is
quadrilateral. AED and EFC are
straight lines.

It is given that x AB 20 = ,
y AE 8 = , y x DC 24 25 = ,
AD AE
4
1
= and EC EF
5
3
= .
(a) Express in terms of x and/or y:
(i) BD
(ii) EC
[3 marks]
(b) Show that the points B, F and D
are collinear.
[3 marks]
(c) If |x| = 2 and |y| = 3, find BD .
[2 marks]
Ans: (a)(i) 20x = 32y, (ii) 25x,
(b) BD = 4BF, (c) 104

O
x
y
B
C
A(4, 9)
2y + x + 6 = 0
O
x
A(0, 6 )
y
B(9, 0)
C
D
F
D
C
B A
E
O
P
B
Q
A
R
53
Program Cemerlang Bersama SMT Zon Johor Program Cemerlang Bersama SMT Zon Johor Program Cemerlang Bersama SMT Zon Johor Program Cemerlang Bersama SMT Zon Johor
2006 2007 2008
9.
Solution of this question by scale drawing will
not be accepted.
Diagram below shows the triangle
AOB where O is the origin. Point C
lies on the straight line AB.

(a) Calculate the area, in unit
2
, of
triangle AOB.
[2 marks]
(b) Given that AC : CB = 3 : 2, find
the coordinate of C.
[2 marks]
(c) A point P moves such that its
distance from point A is always
twice its distance from point B.
i) Find the equation of the locus
of P.
ii) Hence, determine whether or
not this locus intercepts the y-
axis.
[6 marks]
Ans: (a) 9unit2, (b)

5
2
,
5
12
C ,
(c) 3x
2
+3y
2
54x + 24y 135 = 0

2.
In Diagram below, the straight line
AB has an equation y + 2x + 8 = 0. AB
intersects the x-axis at point A and
intersects the y-axis at point B.

Point P lies on AB such that
AP : PB = 1 : 3
Find
(a) the coordinate of P,
[3 marks]
(b) the equation of the straight line
that passes through P and
perpendicular to AB.
[3 marks]
Ans: (a) P(3, 2), (b)
2
1
2
1
= x y
10.
Solution by scale drawing is not accepted.
Diagram below shows a triangle
OPQ. Point S lies on the line PQ.

(a) A point W moves such that its
distance from point S is always
2.5 units. Find the equation of the
locus W.
[3 marks]
(b) It is given that point P and point
Q lie on the locus W. Calculate
i) the value of k,
ii) the coordinates of Q.
[5 marks]
(c) Hence, find the area, in unit
2
, of
triangle OPQ.
[2 marks]
Ans: (a) 4x
2
+4y
2
24x 8y + 15=0
(b)(i) k = 3, (ii)

1 ,
4
9
Q ,
(c)
4
1
5 unit
2

5.
Diagram below shows a trapezium
ABCD.

It is given that y AB 2 = , x AD 6 = ,
AD AE
3
2
= and AD BC
6
5
= .
(a) Express AC , in terms of x and y.
[2 marks]
(b) Point F lies inside the trapezium
ABCD such that AB m EF = 2 ,
and m is a constant.
(i) Express AF , in terms of m, x
and y.
(ii) Hence, if the points A, F and
C are collinear, find the
value of m.
[5 marks]
Ans: (a) 5x + 2y,
(b)(i) 4x + my, (ii)
5
8
= m


8.
Diagram below shows triangle AOB.
The point P lies on OA and the point
Q lies on AB. The straight line BP
intersects the straight line OQ at the
point S.

It is given that OA : OP = 4 : 1,
AB : AQ = 2 : 1. x OA 8 = , y OB 6 = .
(a) Express in terms of x and/or y,
(i) BP
(ii) OQ
(b) Using OQ h OS = and
BP k BS = , where h and k are
constants, find the value of h and
of k.
(c) Given that |x| = 2 units and |y| = 3
units and AOB = 90
0
, find AB .
Ans: (a)(i) 2x 6y (ii) 4x + 3y
(b)
5
4
,
5
2
= = k h , (c) 580
In diagram below, ABCD is a
quadrilateral. The diagonals BD and
AC intersect at point R. Point P lies
on AD.

It is given that AD AP
3
1
= ,
BD BR
3
1
= , x AB = and y AP = .
(a) Express in terms of x and y
(i) DB
(ii) AR
[3 marks]
(b) Given that y x k DC = and
AC h AR = , where h and k are
constants, find the value of h and
of k.
[4 marks]
Ans: (a)(i) x 3y, (ii) y x +
3
2

(b)
3
4
,
2
1
= = k h

O
x
y
S(3, 1)
P

k ,
2
3

Q
O
x
y
P
B
A
y + 2x + 8 = 0
O
x
y
C
B(6, 2)
A(3, 4)
O
P
A
Q
B
S
D
C
B
A
R
P
A
B C
D
E

F


54
Ja0ap ge|j me:e-t, Jeat|aa, )ee-

2003 2004 2005
S
T
A
T
I
S
T
I
C
S

4.
The diagram shows a histogram
which represents the distribution of
the marks obtained by 40 pupils in a
test.
(a) Without using an ogive,
calculate the median mark.
[3 marks]
(b) Calculate the standard
deviation of the distribution.
[4 marks]

14


12


10


8


6


4


2


0.5 10.5 20.5 30.5 40.5 50.5
MARKS
Ans: (a) 24.07, (b) 11.74

P
R
O
B
A
B
I
L
I
T
Y

D
I
S
T
R
I
B
U
T
I
O
N
S

10.
(a) Senior citizens make up 20%
of the population of a
settlement.
i) If 7 people are randomly
chosen from the settlement,
find the probability that at
least two of them are senior
citizen.
ii) If the variance of senior
citizens is 128, what is the
population of the
settlement?
[5 marks]

(b) The mass of the workers in a
factory is normally distributed
with a mean of 67.86kg and a
variance of 42.25kg
2
. 200 of
the workers in the factory
weight between 50kg and
70kg. Find the total number of
worker in the factory.
[5 marks]
Ans: (a)(i) 0.4233, (ii) 800, (b) 319
11.
(a) A club organises a practice
session for trainees on scoring
goals from penalty kicks. Each
trainee takes 8 penalty kicks.
The probability that a trainee
scores a goal from a penalty
kick is p. After the session, it is
found that the mean number of
goals for a trainee is 4.8.
i) Find the value of p.
ii) If a trainee is chosen at
random, find the probability
that he scores at least one
goal.
[5 marks]

(b) A survey on body-mass is done
on a group of students. The
mass of a student has a normal
distribution with a mean of
50kg and a standard deviation
of 15kg.
i) If a student is chosen at
random, calculate the
probability that his mass is
less than 41kg.
ii) Given that 12% of the
students have a mass of
more than m kg, find the
value of m.
[5 marks]
Ans: (a)(i) p = 0.6, (ii) 0.9993
(b)(i) 0.2743, (ii) 67.625kg

11.
For this question, give your answer
correct to three significant figures.

(a) The result of a study shows
that 20% of the pupils in a city
cycle to school. If 8 pupils
from the city are chosen at
random, calculate the
probability that
i) exactly 2 of them cycle to
school,
ii) less than 3 of them cycle to
school.
[4 marks]

(b) The mass of water-melons
produced from an orchard
follows a normal distribution
with a mean of 3.2kg and a
standard deviation of 0.5kg.
Find,
i) the probability that a water-
melon chosen randomly
from the orchard has a mass
of not more than 4.0 kg.
ii) the value of m, if 60% of
the water melons from the
orchard have a mass of
more than m kg.
[6 marks]
Ans: (a)(i) 0.2936, (ii) 0.7969
(b)(i) 0.9452, (ii) m= 3.0735 kg

55
Program Cemerlang Bersama SMT Zon Johor Program Cemerlang Bersama SMT Zon Johor Program Cemerlang Bersama SMT Zon Johor Program Cemerlang Bersama SMT Zon Johor
2006 2007 2008
6.
The table shows the frequency
distribution of the scores of a group
of pupils in a game,
(a) It is given that the median score
of the distribution is 42.
Calculate the value of k.
[3marks]
Score Number of pupils
10 19 1
20 29 2
30 39 8
40 49 12
50 59 k
60 69 1
Use the graph paper to answer this question
(b) Using a scale of 2 cm to 10
scores on the horizontal axis and
2 cm to 2 pupils on the vertical
axis, draw a histogram to
represent the frequency
distribution of the scores.
[4 marks]
(c) What is the mode score if the
score of each pupil is increased
by 5? [3 marks]
Ans: (a) k= 4, (b) mode = 43, (c) 48
5.
Table below shows the cumulative
frequency distribution for the scores
of 32 students in a competition.
Score <10 <20 <30 <40 <50
No of
Students
4 10 20 28 32

(a) Based on table above, copy and
complete table below.

Score
0-9 10-19 20-29 30-39 40-49
No. of
Students

[1 marks]
(b) Without drawing ogive, find the
interquartile range of the
distribution.
[5 marks]
Ans: (b) 18.33

5.
Table below shows the marks
obtained by 40 candidates in a test.
Marks
Number
of candidates
10 19 4
20 29 x
30 39 y
40 49 10
50 59 8

Given that the median marks is 35.5,
find the value of x and y. Hence state
the modal class.
[6 marks]
Ans: x = 12, y = 5, 20 29
11.
An orchard produces lemons. Only
lemons with diameter, x greater than k
cm are graded and marketed.

Grade A B C
Diameter
(cm)
x > 7 7 _ x > 5 5 _ x > k

It is given that the diameter of lemons
has a normal distribution with a mean
of 5.8 cm and a standard deviation of
1.5 cm.
(a) If one lemon is picked at random,
calculate the probability that it is
of grade A..
[2 marks]
(b) In a basket of 500 lemons,
estimate the number of grade B
lemons.
[4 marks]
(c) If 85.7% of the lemons is
marketed, find the value of k.
[4 marks]
Ans: (a) ,(b) ,(c)
11.
(a) In a survey carry out in a school,
it is found that 2 out of 5 students
have hand phones. If 8 students
from that school are chosen at
random, calculate the probability
that
i) exactly 2 students have hand
phones,
ii) more than 2 students have
hand phones.
[5 marks]
(b) A group of workers are given
medical check up. The blood
pressure of a worker has a
normal distribution with a mean
of 130 mmHg is classified as
high blood pressure
i) A worker is chosen at
random from a group. Find
the probability that the
worker has a blood pressure
114 mmHg and 150 mmHg.
ii) It is found that 132 workers
have high blood pressure.
Find the total number of
workers in the group.
[5 marks]
Ans: (a)(i) 0.2090, (ii) 0.6846
(b)(i) 0.7357, (ii) 1250




11.
The mass of mangoes from an
orchard has a normal distribution with
a mean of 300 g and standard
deviation of 80 g.
(a) Find the probability that a mango
chosen randomly from this
orchard has a mass of more than
168 g.
[3 marks]
(b) A random sample of 500
mangoes is chosen.
i) Calculate the number of
mangoes from this sample
that have a mass of more than
168 g.
ii) Given that 435 mangoes from
this sample have a mass of
more than m g, find the value
of m.
[7 marks]
Ans: (a) 0.9595,
(b)(i) 475, (ii) m = 209.84 kg



5B
Ja0ap ge|j me:e-t, Jeat|aa, )ee-

2003 2004 2005
T
R
I
G
O
N
O
M
E
T
R
I
C

F
U
N
C
T
I
O
N
S

8.
(a) Prove that
tan + cot = 3cosec 2
[4 marks]
(b) (i) Sketch the graph
x y
2
3
cos 2 = for 2 0 x .
(ii) Find the equation of a
suitable straight line for solving
the equation 1
4
3
2
3
cos = x x


for 2 0 x . Hence, using
the same axes, sketch the
straight line and state the
number of solutions for the
above equation.
[6 marks]
Ans: (b)(ii) 2
2
3
= x y

,
NOS = 3
3.
(a) Sketch the graph of x y 2 cos =
for 180 0 x .
[3 marks]
(b) Hence, by drawing the suitable
straight line on the same axes,
find the number of solutions
satisfying the equation

=
180
2 sin 2
2
x
x for
180 0 x .
[3 marks]
Ans: (b) 1
180
=
x
y , NOS = 2
5.
(a) Prove that
x x x x ec 2 cos cot sin 2 cos
2 2 2
=
[2 marks]
(b) (i) Sketch the graph of
x y 2 cos = for 2 0 x .
(ii) Hence, using the same axes,
draw the suitable straight line to
find the number of solutions to
the equation
( ) 1 cot sin 2 cos 3
2 2 2
=

x
x x x ec
for 2 0 x . State the
number of solutions.
[6 marks]
Ans: (b)
3
1
3
=

x
y , NOS = 4
L
I
N
E
A
R

L
A
W

7.
Table below shows the values of
two variables, x and y, obtained
from an experiment. Variables x and
y are related by the
equation
2
x
pk y = , where p and k
are constant.

x 1.5 2.0 2.5 3.0 3.5 4.0
y 1.59 1.86 2.40 3.17 4.36 6.76

(a) Plot log y against x
2
, using a
scale 2 cm to 2 unit on the x-
axis and 2 cm to 0.2 unit on the
log y -axis.
Hence, draw the line of best fit.
[5 marks]
(b) Use your graph from (a) to find
the value of
(i) p,
(ii) k.
[5 marks]
Ans: p = 1.259, k = 1.109
7.
Table below shows the values of
two variables, x and y, obtained
from an experiment. The variables x
and y are related by the
equation
x
pk y = , where p and k are
constant.

x 2 4 6 8 10 12
y 3.16 5.50 9.12 16.22 28.84 46.77

(a) Plot log y against x, using a
scale 2 cm to 2 unit on the x-
axis and 2 cm to 0.2 unit on the
log y-axis. Hence, draw the line
of best fit.
[5 marks]
(b) Use your graph from (a) to find
the value of
(i) p,
(ii) k
[5 marks]
Ans: p = 1.820, k = 1.309
7.
Table below shows the values of
two variables, x and y, obtained
from an experiment. The variables x
and y are related by the
equation
px
r
px y + = , where p and
r are constant.

x 1.0 2.0 3.0 4.0 5.0 5.5
y 5.5 4.7 5.0 6.5 7.7 8.4

(a) Plot xy against x
2
, using a scale
2 cm to 5 unit on the x
2
-axis
and 2 cm to 5 unit on the xy -
axis. Hence, draw the line of
best fit.
[5 marks]
(b) Use your graph from 3(a) to
find the value of
(i) p,
(ii) r
[5 marks]
Ans: p = 1.37, r = 5.48

















57
Program Cemerlang Bersama SMT Zon Johor Program Cemerlang Bersama SMT Zon Johor Program Cemerlang Bersama SMT Zon Johor Program Cemerlang Bersama SMT Zon Johor
2006 2007 2008
4.
(a) Sketch the graph of x y cos 2 =
for 2 0 x .
[4 marks]
(b) Hence, using the same axis,
sketch a suitable graph to find the
number of solutions to the
equation 0 cos 2 = + x
x


for 2 0 x .
[3 marks]
Ans:
x
y

= , NOS = 2
3.
(a) Sketch the graph of
x y 2 cos 3 = for 2 0 x .
[4 marks]
(b) Hence, using the same axes,
sketch a suitable straight line to
find the number of solutions for
the equation
2
2 cos 3 2
x
x = for
2 0 x . State the number of
solutions.
[3 marks]
Ans:
2
2
x
y = , NOS = 8
4.
(a) Prove that x
x
x
2 tan
sec 2
tan 2
2
=

.
[2 marks]
(b) (i)Sketch the graph of
x y 2 tan = for x 0 .
(ii) Hence, using the same axes,
sketch a suitable straight line to
find the number of solutions for
the equation 0
sec 2
tan 2 3
2
=

+
x
x x


for x 0 . State the number
of solutions.
[6 marks]
Ans:

x
y
3
= , NOS = 3





7.
Use graph paper to answer this question.
Table below shows the values of two
variables, x and y, obtained from an
experiment. Variables x and y are
related by the equation
1 +
=
x
pk y ,
where p and k are constants.

x 1 2 3 4 5 6
y 4.0 5.7 8.7 13.2 20.0 28.8

(a) Plot log y against (x + 1), using a
scale 2 cm to 1 unit on the (x + 1)-
axis and 2 cm to 0.2 unit on the
log y -axis.
Hence, draw the line of best fit.
[5 marks]
(b) Use your graph from (a) to find
the value of
i) p,
ii) k
[5 marks]
Ans: p = 1.778, k = 1.483
7.
Use graph paper to answer this question.
Table below shows the values of two
variables, x and y, obtained from an
experiment. Variables x and y are
related by the
equation x
k
p
kx y + =
2
2 , where p and
k are constants.

x 2 3 4 5 6 7
y 8 13.2 20 27.5 36.6 45.5

(a) Plot
x
y
against x, using a scale of
2 cm to 1 unit on both axes.
Hence draw the line of best fit.
[4 marks]
(b) Use your graph in (a) to find the
value of
(i) p,
(ii) k.
(iii) y when x = 1.2
[6 marks]
Ans: p = 0.754, k = 0.26, y = 4.2
8.
Use graph paper to answer this question.
Table below shows the values of two
variables, x and y, obtained from an
experiment. Variables x and y are
related by the equation y = hk
2x
,
where h and k are constants.

x 1.5 3.0 4.5 6.0 7.5 9.0
y 2.51 3.24 4.37 5.75 7.76 10.00

(a) Based on table above, construct a
table for the values of log
10
y.
[1 mark]
(b) Plot log
10
y against x, using a
scale of 2 cm to 1 unit on the x-
axis and 2 cm to 0.1 unit on the
log
10
y-axis. Hence draw the line
of best fit.
[4 marks]
(c) Use your graph in (b) to find the
value of
(i) x when y = 4.8
(ii) h,
(iii) k.
[5 marks]
Ans: x = 5, h = 1.91, k = 1.094















5B
Ja0ap ge|j me:e-t, Jeat|aa, )ee-

2003 2004 2005
C
I
R
C
U
L
A
R

M
E
A
S
U
R
E
S

4.
Diagram below shows the sector
POQ, centre O with radius 10 cm.
The point R on OP such that
OR : OP = 3 : 5.

Calculate
(a) the value of , in rad.
[3 marks]
(b) the area of shaded region, in
cm
2
.
[4 marks]
Ans: (a) 0.9274 rad., (b) 22.37 cm
2


9.
Diagram below shows a circle
PQRT, centre O and radius 5 cm.
JQK is a tangent to the circle at Q.
The straight lines, JO and KO,
intersect the circle at P and R
respectively. OPQR is a rhombus.
JLK is and arc of a circle, centre O.

Calculate
(a) the angle, , in terms of ,
[2 marks]
(b) the length, in cm, of the arc
JLK,
[4 marks]
(c) the area, in cm
2
, of the shaded
region.
[4 marks]
Ans: (a) .
3
2
rad , (b) 20.94 cm,
(c) 61.40 cm
2

10.
Diagram shows a sector POQ of a
circle with centre O. Point A lies on
OP, point B lies on OQ and AB is
perpendicular to OQ. The length of
OA is 8 cm and POQ =
6

radians.

It is given that OA : OP = 4 : 7
(Use a = 3.142)
Calculate
(a) the length, in cm, of AP,
[1 marks]
(b) the perimeter, in cm, of the
shaded region,
[5 marks]
(c) the area, in cm
2
, of the shaded
region.
[4 marks]
Ans: (a) 6 cm, (b)24.403 cm,
(c) 37.46 cm
2



































O
u rad
P
Q
R
L
T
O
R
P
Q
O
B Q
P
A
8 cm
59
Program Cemerlang Bersama SMT Zon Johor Program Cemerlang Bersama SMT Zon Johor Program Cemerlang Bersama SMT Zon Johor Program Cemerlang Bersama SMT Zon Johor
2006 2007 2008
10.
Diagram below shows the plan of a
garden. PCQ is a semicircle with
centre O and has a radius of 8 m.
RAQ is sector of a circle with centre A
and has a radius of 14 m.

Sector COQ is a lawn. The shaded
region is a flower bed and has to be
fenced. It is given that AC = 8 m and
COQ = 1.956 radians.
[Use a = 3.142]
Calculate
(a) the area, in m
2
, of the lawn,
[2 marks]
(b) the length, in m, of the fence
required for fencing the flower
bed,
[4 marks]
(c) the area, in m
2
, of the flower bed.
[4 marks]
Ans: (a) 62.592m
2
, (b) 38.252m,
(c) 31.363 m
2

9.
Diagram below shows a circle, centre
O and radius 10 cm inscribed in a
sector APB of a circle, centre P. The
straight lines, AP and BP, are tangents
to the circle at point Q and point R,
respectively.

[Use a = 3.142]
Calculate
(a) the length, in cm, of the arc AB,
[5 marks]
(b) the area, in cm
2
, of the shaded
region.
[5 marks]
Ans: (a) 31.42 cm, (b) 88.64 cm
2

9.
Diagram below shows two circles.
The larger circle has centre X and
radius 12cm. The smaller circle has
centre Y and radius 8 cm. The circle
touches at point R. The straight line
PQ is a common tangent to the circles
at point P and point Q.

[Use a = 3.142]
Given that PXR = radians,
(a) Show that = 1.37 (to two
decimal places),
[ marks]
(b) calculate the length, in cm, of the
minor arc QR,
[ marks]
(c) calculate the area, in cm
2
, of the
coloured region.
[ marks]
Ans: (b) 14.179cm, (c) 40.68 cm
2



































8 cm
X
Y
R
P
Q
12 cm
60

O

A

B

P

R

10 cm

Q

P
C
A O Q
R

Bo
Ja0ap ge|j me:e-t, Jeat|aa, )ee-

2003 2004 2005
D
I
F
F
E
R
E
N
T
I
A
T
I
O
N

&

I
N
T
E
G
R
A
T
I
O
N

3.
(a) Given that 2 2 + = x
dx
dy
and
y=6 when x = 1, find y in
terms of x.
[3 marks]
(b) Hence, find the value of x if
8 ) 1 (
2
2
2
= + + y
dx
dy
x
dx
y d
x
[4 marks]
Ans: (a) y = x
2
+ 2x + 7
(b) 1 ,
5
3
= x

9(b)
(b) Diagram below shows a curve
1
2
= y x which intersects the
straight line 3y = 2x at point A.

Calculate the volume generated
when the shaded region is
revolved 360
0
about the y-axis
[6 marks]
Ans:
15
52

5.
The gradient function of a curve
which passes through A(1, -12) is
3x
2
6x.
Find
(a) the equation of the curve
[3 marks]
(b) the coordinates of the turning
points of the curve and
determine whether each of the
turning points is a maximum or
minimum.
[5 marks]
Ans: (a) y = 3x
2
6x 10,
(b) (0, 10) Max, (2, 10)Min


10.
Diagram below shows part of the
curve
2
) 1 2 (
3

=
x
y which passes
through A(1, 3).

(a) Find the equation of tangent to
the curve at the point A.
[4 marks]
(b) A region is bounded by the
curve, the x-axis and the
straight lines
x = 2 and x = 3.
(i) Find the area of the region.
(ii) The region is revolved
through 360
0
about the x-
axis. Find the volume
generated, in terms of .
[6 marks]
Ans: (a) y = 12x + 15
(b)(i)
2
5
1
unit , (ii)
3
1125
49
unit




2.
A curve has a gradient
function x px 4
2
, where p is a
constant. The tangent to the curve at
the point (1, 3) is parallel to the
straight line y + x 5 = 0.
Find
(a) the value of p,
[3 marks]
(b) the equation of the curve.
[3 marks]
Ans: (a) p = 3, (b) f(x)=x
3
2x
2
+3


8.
In diagram below, the straight line
PQ is normal to the curve
1
2
2
+ =
x
y at A(2, 3). The straight
line AR is parallel to the y-axis.


Find
(a) the value of k,
[2 marks]
(b) the area of shaded region,
[5 marks]
(c) the volume generated, in term
of , when the region bounded
by the curve, the y-axis and the
straight line y = 3 is revolved
through 360
0
about the y-axis.
[3 marks]
Ans: (a) k = 8, (b)
2
3
1
12 unit ,
(c) 4 unit
3















A(2, 3)
1
2
2
+ =
x
y

O
y
x
P
Q(k, 0)
x = y
2
1
y
x
A 1
O
1
3y = 2x
A(1, 3)
2
) 1 2 (
3

=
x
y

O
y
x
B1
Program Cemerlang Bersama SMT Zon Johor Program Cemerlang Bersama SMT Zon Johor Program Cemerlang Bersama SMT Zon Johor Program Cemerlang Bersama SMT Zon Johor
2006 2007 2008
8.
Diagram below shows the straight
line 4 + = x y intersecting the curve
2
) 2 ( = x y at the points A and B.

Find
(a) the value of k,
[3 marks]
(b) the area of the shaded region P,
[5 marks]
(c) the volume generated, in terms of
, when the shaded region Q is
revolved 360 about the x-axis.
[3 marks]
Ans: (a) k = 5, (b) 20.83 unit2,
(c)
3
5
32
unit
4.
A curve with gradient
function
2
2
2
x
x , has a turning point
at (k, 8).
(a) Find the value of k.
[3 marks]
(b) Determine whether the turning
point is a maximum or minimum
point.
[2 marks]
(c) Find the equation of the curve.
[3 marks]
Ans: (a) k = 1, (b) Min(1, 8),
(c) 5
2
2
+ + =
x
x y

10.
Diagram below shows part of the
curve
3
) 1 ( = x k y , where k is a
constant.

The curve intersects the straight line
x = 3 at point A. At point A, 24 =
dx
dy
.
(a) Find the value of k
[3 marks]
(b) Hence, calculate
i) the area of the shaded region
P,
ii) the volume generated, in
terms of a, when the region R
which is bounded by the
curve, the x-axis and y-axis,
is revolved through 360
0

about x-axis.
[7 marks]
Ans: (a) k = 2,
(b)(i) 8 unit2, (ii)
2
49
4
unit


Diagram below shows the curve
5
2
+ = x y and the tangent to the
curve at the point A(1, 6).

Calculate
(a) the equation of the tangent at A,
[3 marks]
(b) the area of the coloured region,
[4 marks]
(c) the volume of revolution, in
terms of , when the region
bounded by the curve and the
straight line y = 7 is rotated
through 360
0
about the y-axis.
[3 marks]
Ans: (a) y = 2x = 4, (b)
2
3
1
unit ,
(c) 2 unit
3

















y
x
y = x
2
+ 5

A(1, 6)
O
p
A
R
y
x
O
x = 3
y = k(x 1)
3
y = x + 4
y = (x 2)
2
A
P
k
O
Q
x
y

B2
Ja0ap ge|j me:e-t, Jeat|aa, )ee-
SECTION C SECTION C SECTION C SECTION C


2003 2004 2005
I
N
D
E
X

N
U
M
B
E
R
S

13.
Diagram shows a bar chart
indicating the weekly cost of the
item P, Q, R, S and T for the year
1990. Table below shows the prices
and the price indices for the items.
Items Price in
1990
(RM)
Price in
1995
(RM)
Price
index in
1995
based on
1990
P x 0.70 175
Q 2.00 2.50 125
R 4.00 5.50 y
S 6.00 9.00 150
T 2.50 z 120

(a) Find the value of
(i) x,
(ii) y,
(iii) z.
[3 marks]
(b) Calculate the composite index
for the items in the year 1995
based on the year 1990.
[2 marks]
(c) The total monthly cost of the
items in the year 1990 is
RM456. Calculate the
corresponding total monthly
cost for the year 1995.
[2 marks]
(d) The cost of the items increases
by 20% from the year 1995 to
the year 2000. Find the
composite index for the year
2000 based on the year 1990.
[3marks]
Ans: (a) x = 0.40, y = 137.5, z = 3
(b) 140.9, (c) RM642.50, (d) 169.1
12.
Table below shows the price indices
and percentage of usage of four
items, P, Q, R and S, which are the
main ingredients in the production
of a type of biscuit.
Items Price index for
the year 1995
based on the
year 1993
Percentage of
usage (%)
P 135 40
Q x 30
R 105 10
S 130 20

(a) Calculate
i) the price of S in the year
1993 if its price in the year
1995 is RM37.70.
ii) the price index of P in the
year 1995 based on the year
1991 if its price index in the
year 1993 based on the year
1991 is 120.
[5 marks]
(b) The composite index number of
the lost of biscuit production of
the year 1995 based on the year
1993 is 128. Calculate
i) the value of x,
ii) the price of a box of biscuit
in the year 1993 if the
corresponding price in the
year 1995 is RM32.
[5 marks]
Ans: (a)(i) RM29, (ii) 162
(b)(i) x = 125, (ii) RM25
13.
Table below shows the prices and
the price indices for the four
ingredients, P, Q, R and S, used in
making biscuits of a particular kind.
Diagram shows a pie chart which
represents the relative amount of the
ingredients P, Q, R and S used in
making biscuits.

(a) Find the value of x, y and z.
[3 marks]
(b) (i) Calculate the composite
index for the cost of making
these biscuits in the year 2004
based on the year 2001.
(ii) Hence, calculate the
corresponding cost of making
these biscuits in the year 2001 if
the cost in the year 2004 was
RM2,985.
[5 marks]
(c) The cost of making these
biscuits is expected to increase
by 50% from the year 2004 to
the year 2007. Find the
expected composite index for
the year 2007 based on the year
2001.
[2 marks]
Ans: (a) x = 125, y=2.80, z = 0.50
(b)(i) 129.44, (ii)RM2306.09,
(c) 194.16

Price per kilogram
(RM)
I
n
g
r
e
d
i
e
n
t
s

Year
2001
Year
2004
Price index
for the year
2004 based on
the year 2001
P 0.80 1.00 x
Q 2.00 y 140
R 0.40 0.60 150
S z 0.40 80













100
R
Q
P
S
60
120
P Q R S T
12
15
24
30
33
Weekly cost (RM)
B3
Program Cemerlang Bersama SMT Zon Johor Program Cemerlang Bersama SMT Zon Johor Program Cemerlang Bersama SMT Zon Johor Program Cemerlang Bersama SMT Zon Johor


2006 2007 2008
15.
A particular kind of cake is made by
using four ingredients, P, Q, R and S.
Table 5 shows the prices of the
ingredients.
Price per kg Ingredient
Year 2004 Year 2005
P 5.00 w
Q 2.50 4.00
R x y
S 4.00 4.40

(a) The index number of ingredient
P in the year 2005 based on the
year 2004 is 120. Calculate w.
[2 marks]
(b) The index number of ingredient
R in the year 2005 based on the
year 2004 is 125. The price per
kilogram of ingredient R in the
year 2005 is RM2.00 more than
its corresponding price in the
year 2004. Calculate the value of
x and y.
[3 marks]
(c) The composite index for the cost
of making the cake in the year
2005 based on the year 2004 is
127.5. Calculate;
(i) the price of a cake in the year
2004 if its corresponding price
in the year 2005 is RM30.60.
(ii) the value of m, if the quantities
of ingredient P, Q, R and S used
in the ratio of 7 : 3 : m : 2.
[5 marks]
Ans: (a) w = 6, (b) x = 8, y = 10,
(c)(i) RM24, (ii) m = 4
13.
Table below shows the prices and the
price indices of five components, P,
Q, R, S and T, used to produce a kind
of toy. Pie cart represents the relative
quantity of components used.


(a) Find the value of x and y.
[3 marks]
(b) Calculate the composite index for
the year 2006 based on the year
2004.
[3 marks]
(c) This price of each component
increases by 20%from the year
2006 to the year 2008.
Given that the production cost of
one toy in the year 2004 is
RM55, calculate the
corresponding cost in the year
2008.
[4 marks]
Ans: (a) x = 2.00, y = 90, (b) 123.5
(c) RM81.51


Price (RM) for
the year
C
o
m
p
o
n
e
n
t

Year
2004
Year
2006
Price index for
the year 2006
based on the
year 2004
P 1.20 1.50 125
Q x 2.20 110
R 4.00 6.00 150
S 3.00 2.70 y
T 2.00 2.80 140
13.
Table below shows the prices and
price indices of four ingredients, fish,
flour, salt and sugar, used to make a
type of fish cracker. Pie cart
represents the relative quantity of the
ingredients used.

(a) Find the value of h and k.
[3 marks]
(b) Calculate the composite index for
the cost of making these crackers
in the year 2005 based on the
year 2004.
[3 marks]
(c) The composite index for the cost
of making these crackers
increases by 50% from the year
2005 to the year 2009. Calculate
i) the composite index for the
cost of making these crackers
in the year 2009 based on the
year 2004,
ii) the price of a box of these
crackers in the year 2009 if
its corresponding price in the
year 2004 is RM25.
[4 marks]
Ans: (a) h = 120, k = 0.80,
(b) 126.38,
(c)(i) 189.57, (ii) RM47.39


Price (RM) per
kg of the year
I
n
g
r
e
d
i
e
n
t
s

Year
2004
Year
2005
Price index
for the year
2005 based on
the year 2004
Fish 3.00 4.50 150
Flour 1.50 1.80 h
Salt k 0.90 112.5
Sugar 1.40 1.47 105














36
0
144
0
72
0
P

Q

R
S

T

1
Q
P
S 1
30%

45%

15%
Sugar

Flour

Fish

10%

Salt


B4
Ja0ap ge|j me:e-t, Jeat|aa, )ee-

2003 2004 2005
S
O
L
U
T
I
O
N

O
F

T
R
I
A
N
G
L
E
S

15.
Diagram below shows a tent VABC
in the shape of pyramid with
triangle ABC as the horizontal base.
V is the vertex of the tent and the
angle between the inclined plane
VBC and the base is 50
0


Given that VB = VC = 2.2 m and
AB = AC = 2.6 m, calculate
(a) the length of BC if the area of
the base is 3 m
2
,
[3 marks]
(b) the length of AV if the angle
between AV and the base is 25,
[3 marks]
(c) the area of triangle VAB.
[4 marks]
Ans: (a) BC = 2.7m,
(b) AV = 3.149m, (c) 2.829m
2

13.
Diagram below shows a
quadrilateral ABCD such that ABC
is acute.

(a) Calculate
(i) ABC,
(ii) ADC,
(iii) the area, in cm
2
, of
quadrilateral ABCD.
[8 marks]
(b) A triangle, ABC has the same
measurements as those given
for triangle ABC, that is, AC
= 12.3cm, CB = 9.5 cm and
BAC = 40.5

, but which is
different in shape to triangle
ABC.
(i) Sketch the triangle ABC,
(ii) State the size of ABC.
[2 marks]
Ans: (a)(i) 57.23, (ii) 106.07,
(iii) 80.96cm
2
, (b)(ii) 122.77
12.
Diagram below shows triangle ABC.

(a) Calculate the length, in cm, of
AC.
[2 marks]
(b) A quadrilateral ABCD is now
formed so that AC is a
diagonal, ACD = 40


and AD
= 16 cm, Calculate the two
possible values of ADC.
[2 marks]
(c) By using the acute ADC from
(b), calculate
(i) the length, in cm, of CD,
(ii) the area, in cm
2
, of the
quadrilateral ABCD.
[6 marks]
Ans: (a) AC = 19.27cm,
(b) 50.73 or 129.27
(c)(i) 24.89cm, (ii) 290.1 cm
2

M
O
T
I
O
N

A
L
O
N
G

A

S
T
R
A
I
G
H
T

L
I
N
E

12.
A particle moves in a straight line
and passes through a fixed point O,
with a velocity of 24 ms
-1
. Its
acceleration, a ms
-2
, t s after passing
through O is given by a = 10 2t.
The particle stops after k s.
(a) Find
(i) the maximum velocity of the
particle,
(ii) the value of k.
[6 marks]
(b) Sketch a velocity-time graph
for 0 _ t _ k.
Hence, or otherwise, calculate
the total distance traveled
during that period.
[4 marks]
Ans: (a)(i) 49, (ii) k = 12, (b)432m
15.
A particle moves along a straight
line from a fixed point P. Its
velocity v ms
-1
, is given by
V = 2t(6 t), where t is the time, in
seconds, after leaving the point P.
[Assume motion to the right is
positive]
Find
(a) the maximum velocity of the
particle,
[3 marks]
(b) the distance traveled during the
third second,
[3 marks]
(c) the value of t when the particle
passes the point P again,
[2 marks]
(d) the time between leaving P and
when the particle reverses its
direction of motion.
[2 marks]
Ans: (a) 18ms
-1
, (b) m
3
1
17 ,
(c) t = 9, (d) 6 s.
15.
Diagram below shows the positions
and directions of motion of two
objects, P and Q, moving in a
straight passing two fixed point A
and B respectively. Object P passes
the fixed point A and object Q
passes the fixed point B
simultaneously. The distance AB is
28 m.

The velocity of P, v
p
m s
-1
, is given
v
p
= 6 + 4t 2t
2
, where t is the time,
in seconds A, after it passes A while
Q travels with a constant velocity of
2 ms
-1
. Object P stops
instantaneously at the point C.
[Assume that the positive direction
of motion is towards the right]
Find
(a) the maximum velocity, in ms
-1
,
of P,
[3 marks]
(b) the distance, in m, of C from A,
[4 marks]
(c) the distance, in m, between P
and Q when P is at the point C.
[3 marks]
Ans: (a) 8ms
-1
, (b) 18m, (c) 4m

P Q
A C B
28 m
65
20 cm
A
B
C
15 cm
A
B
C
D
12.3 cm
9.8 cm
5.2 cm
9.5 cm
40.5
A
B
C
V
B5
Program Cemerlang Bersama SMT Zon Johor Program Cemerlang Bersama SMT Zon Johor Program Cemerlang Bersama SMT Zon Johor Program Cemerlang Bersama SMT Zon Johor
2006 2007 2008
13.
Diagram below shows quadrilateral
ABCD.

The area of triangle BCD is 13 cm
2

and BCD is acute. Calculate
(a) BCD
[2 marks]
(b) the length, in cm, of BD
[2 marks]
(c) ABD
[3 marks]
(d) the area, in cm
2
, quadrilateral
ABCD.
[3 marks]
Ans: (a) 60.07, (b) 5.573cm,
(c) 116.55, (d) 35.43cm
2

15.
Diagram below shows quadrilateral
ABCD.

(a) Calculate
(i) the length, in cm, of AC,
(ii) ACB.
[4 marks]
(b) Point A lies on AC such that AB
= AB.
(i) Sketch ABC.
(ii) Calculate the area, in cm
2
, of
ABC.
[6 marks]
Ans: (a)(i) AC=13.36 cm, (ii) 23.88
(b) (ii) 13.80 cm
2

14.
In diagram below, ABC is a triangle.
ADFB, AEC and BGC are straight
lines. The straight line FG is
perpendicular to BC.

It is given that BD = 19 cm, DA = 16
cm, AE = 14 cm, DAE = 80 and
FBG = 45.
(a) Calculate the length, in cm, of
(i) DE,
(ii) EC.
[5 marks]
(b) The area of triangle DAE is twice
the area of triangle FBG.
Calculate the length, in cm, of
BG.
[4 marks]
(c) Sketch triangle ABC which has
a different shape from triangle
ABC such that AB = AB, AC =
AC and ABC = ABC.
[1 mark]
Ans: (a)(i)19.34cm, (ii)16.213cm,
(b) 10.5023cm
2

12.
A particle moves in a straight line and
passes through a fixed point O. Its
velocity, v ms-1, is given by
5 6
2
+ = t t v , where t is the time, in
seconds, after leaving O.
[Assume motion to the right is
positive]
(a) Find
(i) the initial velocity of the
particle,
(ii) the time interval during
which the particle moves towards
the left,
(iii) the time interval during
which the acceleration of the
particle is positive.
[5 marks]
(b) Sketch the velocity-time graph of
the motion of the particle
0 _ t _ 5.
[2 marks]
(c) Calculate the total distance
traveled during the first 5
seconds after leaving O.
[3 marks]
Ans: (a)(i) 5ms-1 (ii) 1<t<5, (c) 13m



12.
A particle moves along a straight line
and passes through a fixed point O.
Its velocity, v ms
-1
, is given by v = t
2

6t + 8, where t is the time, in
second, after passing through O.
[Assume motion to the right is
positive]
Find
(a) the initial velocity, in ms
-1
,
[1 marks]
(b) the minimum velocity, in ms
-1
,
[3 marks]
(c) the range of values of t during
which the particle moves to the
left,
[2 marks]
(d) the total distance, in m, traveled
by the particle in the first 4
seconds.
[4 marks]
Ans: (a) 8ms
-1
, (b) v
min
= 1ms
-1
,
(c) 2 < t < 4, (d) 8m
12.
A particle moves along a straight line
and passes through a fixed point O.
Its velocity, v ms
-1
, is given by
v = 10 + 3t t
2
, where t is the time,
in second, after passing through O.
The particle stops instantaneously at a
point R.
Find
(a) the acceleration, in ms
-2
, of the
particle R,
[3 marks]
(b) the maximum velocity, in ms
-1
,
of the particle.
[3 marks]
(c) the total distance, in m, traveled
by the particle in the first 9
seconds, after passing through O.
[4 marks]
Ans: (a) 7 ms
-1
, (b) 12.25ms
-1
,
(c) m
6
1
123

45

80

C
B
G
A
D
E
F
50

105

A
C D
B
16.4 cm
5.6 cm
6 cm
40
5 cm
6
cm
9 cm
A
B
C
D

BB
Ja0ap ge|j me:e-t, Jeat|aa, )ee-

2003 2004 2005
L
I
N
E
A
R

P
R
O
G
R
A
M
M
I
N
G

14.
Yahya has an allocation of RM225
to buy x kg of prawns and y kg fish.
The total mass of commodities is
not less than 15kg. The mass of the
prawns at the most is three times
that of fish. The price of 1 kg of
prawns is RM9 and the price of 1 kg
of fish is RM5.
(a) Write down three inequalities,
other than x _ 0 and y _ 0,
which satisfy all the above
conditions.
[3 marks]
(b) Hence, using a scale of 2cm to
5kg for both axes, construct and
shade the region R which
satisfies all the above
conditions.
[4 marks]
(c) If Yahya buys 10kg of fish,
what is the maximum amount
of money that could remain
from his allocation?
[3 marks]
Ans: (c)RM130

14.
The district education office intends
to organize a course for the teaching
of Mathematics and Science in
English. The course will be attended
by x Mathematics participants and y
Science participants. The selection
of participants is based on the
following constraints.
I The total number of participants
must be at least 40.
II The number of Science
participants must be at most
twice the number of
Mathematics.
III The maximum allocation for the
course is RM7,200. The
expenditure for a Mathematics
participant is RM120 and for a
Science participant is RM80.
a) Write down three inequalities
other than x _ 0 and y _ 0,
which satisfy all the above
constraints.
[3 marks]
b) Hence, by using the scale of
2cm to 10 participants on both
axes, construct and shade the
region R which satisfy all the
above constraints.
[3 marks]
c) Using your graph from (b),
find;
(i) the maximum and minimum
number of Mathematics
participants when the number
of Science participant is 10.
(ii) the minimum cost needed to
run the course.
[4 marks]
Ans: (c)(i) x
min
= 30, x
max
= 53,
(ii) RM3760





14.
An institution offers two computer
courses, P and Q. the number of
participants of course P is x and for
course Q is y. The enrolment of the
participants is based on the
following constraints:
I. The total number is not more
than 100.
II. The number of participants for
course Q is not more than 4
times the number of
participants for course P.
III. The number of participants for
course Q must exceed the
number of participants for
course P by at least 5.
a) Write down three inequalities
other than x _ 0 and y _ 0,
which satisfy all the above
constraints.
[3 marks]
b) Using the scale of 2cm to 10
participants on both axes,
construct and shade the region
R which satisfy all the above
constraints.
[3 marks]
c) By using your graph from (b),
find;
i) the range of the number of
participants for course Q if
the number of participants for
course P is 30.
ii) the maximum total fees per
month that can be collected if
the fees per month for
courses P and Q are RM50
and RM 60 respectively.
[4 marks]
Ans: (c)(i) 35 < y < 70, (ii)RM5800




















B7
Program Cemerlang Bersama SMT Zon Johor Program Cemerlang Bersama SMT Zon Johor Program Cemerlang Bersama SMT Zon Johor Program Cemerlang Bersama SMT Zon Johor
2006 2007 2008
14.
A workshop produces two types of
racks, P and Q, The production of
each type of rack involves two
processes, making and painting. Table
below shows the time taken to make
and paint a rack pf type P and a rack
of type Q.

Time taken
(minutes)
Rack Making Painting
P 60 30
Q 20 40

The workshop produces x racks of
type P and y racks of type Q per day.
The production of racks per day is
based on the following constraints:
I The maximum total time for
making both racks is 720 minutes.
II The total time for painting both
racks is at least 360 minutes.
III The ratio of the number of racks
of type P to the number of racks
of type Q is at least 1 : 3.
(a) Write down three inequalities
other than x_0 and y_0, which
satisfy all the above constraints.
[3 marks]
(b) Using the scale of 2cm to 2 racks
on both axes, construct and shade
the region R which satisfy all the
above constraints.
[3 marks]
(c) By using your graph from (b),
find;
i) the minimum number of racks
of type Q if 7 racks of type P are
produced per day,
ii) the maximum total profit per
day if the profit from one rack of
type P is RM24 and from one
rack of type Q is RM32.
[4 marks]
Ans: (c)(i) ymin = 4,
(ii) Profit
max
=RM720
14.
A factory produces two components
P and Q. In a particular day, the
factory produced x pieces of
component P and y pieces of
component Q. The profit from the
sales of a piece of component P is
RM15 and a piece of component Q is
RM12.
I The total number of components
produced is at most 500.
II The number of component P
produced is not more than three
times the number of component Q.
III The minimum total profit for both
components is RM4 200.
(a) Write down three inequalities
other than x_0 and y_0, which
satisfy all the above constraints.
[3 marks]
(b) Using the scale of 2cm to 50
components on both axes,
construct and shade the region R
which satisfy all the above
constraints.
[3 marks]
(c) By using your graph from (b),
find;
i) the minimum number pieces of
component Q if the number of
pieces of component P produced
on particular day 100,
ii) the maximum total profit per
day.
[4 marks]
Ans: (c)(i) Q
min
= 225,
(ii) Profit
max
= RM7125
15.
The number of youth association plan
to organize a picnic. They agree to
rent x buses and y vans. The rental of
a bus is RM800 and the rental of a
van is RM300.
The rental of the vehicle for the
picnic is based on the following
constraints:
I The total of vehicles to be rented
is not more than 8.
II The number of buses is at most
twice the number of vans.
III The maximum allocation for the
rental of the vehicles is
RM4,000.
(a) Write down three inequalities
other than x_0 and y_0, which
satisfy all the above constraints.
[3 marks]
(b) Using the scale of 2cm to 1
vehicle on both axes, construct
and shade the region R which
satisfy all the above constraints.
[3 marks]
(c) By using your graph from (b),
find;
i) the minimum number of vans
rented if 3 buses are rented,
ii) the maximum number of
members that can be
accommodated into the
rented vehicles if a bus can
accommodate 48 passengers
and a van accommodate 12
passengers.
[4 marks]
Ans: (c)(i) Van
min
= 2,
(ii) Members
max
= 216

















BB
Ja0ap ge|j me:e-t, Jeat|aa, )ee-

MODULES MODULES MODULES MODULES
"EXPERIENCE IS A GREAT "EXPERIENCE IS A GREAT "EXPERIENCE IS A GREAT "EXPERIENCE IS A GREAT
TEACHER" TEACHER" TEACHER" TEACHER"
B9
Program Cemerlang Bersama SMT Zon Johor Program Cemerlang Bersama SMT Zon Johor Program Cemerlang Bersama SMT Zon Johor Program Cemerlang Bersama SMT Zon Johor
MODULE 1 MODULE 1 MODULE 1 MODULE 1
TRIAL 2007 (MRSM), PAPER 1 TRIAL 2007 (MRSM), PAPER 1 TRIAL 2007 (MRSM), PAPER 1 TRIAL 2007 (MRSM), PAPER 1

1

Diagram 1
Diagram 1 shows the function f : x y.
State
(a) the domain,
(b) the range,
of the function.

[2 marks]

2
It is given that 4 :
1
+

x x g and
2
) 3 ( : x x f
Find
(a) g,
(b) fg.
[3 marks]






3 Solve the quadratic equation 4x
2
9 = (2x + 1)(x 3).
Give your answer correct to three significant figures.
[3 marks]









4 Find the range of the values of x for (x + 1)(x 2) _ 2(x + 4).
[3 marks]














7o
Ja0ap ge|j me:e-t, Jeat|aa, )ee-
5 Solve the equation 8 4
a+1
= 16
2a
.
[3 marks]










6 Solve the equation 1 ) 6 ( log 2 log 3 4 log
3 3 3
= + x x .
[4 marks]










7
Express the equation 3 log log
16 4
= x y in the form y = ax
b
, where a and b are constants.
[4 marks]













8
The sum of the first n terms of an arithmetic progression, S
n
, is given by n n S
n
3 5
2
= . Find
the 5
th
term of the progression.
[3 marks]










71
Program Cemerlang Bersama SMT Zon Johor Program Cemerlang Bersama SMT Zon Johor Program Cemerlang Bersama SMT Zon Johor Program Cemerlang Bersama SMT Zon Johor
9 For a geometric progression, the sum of the first two terms is 30 and the third term exceeds the
first term by 15. Find the common ratio and the first term of the geometric progression.
[4 marks]










10 It is given that p = 0.160606... = 0.1 + q, where q is a recurring decimal. Express p as a
fraction in its simplest form.
[4 marks]











11 Given that the straight line py + x + 5 = 0 is perpendicular to the straight line 2y q
2
x = 6,
where p and q are constants, find the relation between p and q.
[3 marks]









12

Diagram 2
Diagram 2 shows the graph of xy
against x
2
.
The variables x and y are related by the
equation
x
k
x y + = 2 , where k is a
constant.
Find the value of h and of k.
[3 marks]







72
Ja0ap ge|j me:e-t, Jeat|aa, )ee-
13

Diagram 3
Diagram 3 shows a triangle OAB,
where O is the origin.
It is given that the coordinates of point
A and point B are (3,4) and (5,2)
respectively.
Find
(a) AB ,
(b) the possible unit vectors parallel to
OA .
[4 marks]





14 It is given that a and b are non-parallel vector. If (m 1) = (n + 2) , where m and n are
constants, find the value of m and of n.
[2 marks]






15

Diagram 4
Diagram 4 shows the sector POQ of a circle centre O
and radius 10 cm.
It is given that the length of the chord PQ is 12 cm.
Find the area, in cm
2
, of the shaded segment.
[4 marks]











16 Solve the equation 2sin + 3cos = 0 for 0 _ _ 360.
[3 marks]











73
Program Cemerlang Bersama SMT Zon Johor Program Cemerlang Bersama SMT Zon Johor Program Cemerlang Bersama SMT Zon Johor Program Cemerlang Bersama SMT Zon Johor
17
If
t
t
t f
2 5
4 3
) (
2

+
= find f (t).
[3 marks]







18
Find the coordinates of the turning points of the curve .
48
12
3
x
x y =
[4 marks]







19
Given that

=
3
1
5 ) ( dx x f , find the value of [ ]


1
3
. 7 ) ( dx x f
[2 marks]






20
Given that ) ( 2
1 2
1
2
x f
x
x
dx
d
=

, find the value of dx x f

2
0
) ( .
[3 marks]










74
Ja0ap ge|j me:e-t, Jeat|aa, )ee-
21

Diagram 5
Diagram 5 shows part of the curve y = (x 2)(6
x).
Find the area of the shaded region.
[3 marks]













22 Diagram 6 shows nine letter cards to be arranged in a row.

Diagram 6

Calculate the number of different arrangements of all the letter cards if
(a) the arrangement start with vowel,
(b) all the consonants must be together.
[4 marks]





23 A quiz team consists of 6 students. These students are to be chosen from 20 students. Calculate
the number of different ways the team can be formed if
(a) there is no restriction,
(b) 2 particular students must be chosen.
[3 marks]






24 A bag contains 8 cards where 3 of the cards are yellow. Three cards are drawn at random from
the bag, one after the other without replacement. Calculate the probability that at least two
yellow cards are drawn.
[3 marks]






75
Program Cemerlang Bersama SMT Zon Johor Program Cemerlang Bersama SMT Zon Johor Program Cemerlang Bersama SMT Zon Johor Program Cemerlang Bersama SMT Zon Johor
25 The diameters of the marbles produced by a factory are normally distributed with a mean of 9
mm and a standard deviation of 0.1 mm. Diagram 7 shows the normal distribution graph for
the diameter of the marbles, X mm.

Diagram 7
It is given that the area of the shaded region is 0.4522. Find the value of h.
[3 marks]


























END OF QUESTIONS








h

7B
Ja0ap ge|j me:e-t, Jeat|aa, )ee-
M MM MODULE ODULE ODULE ODULE 2 22 2
TRIAL 2007 ( TRIAL 2007 ( TRIAL 2007 ( TRIAL 2007 (PAHANG PAHANG PAHANG PAHANG), PAPER 1 ), PAPER 1 ), PAPER 1 ), PAPER 1

1


Based on the above information, the relation between M and N is defined by the set of ordered
pairs {(2, 1), (1, 0 ), ( 0, 1 ), ( 1, 2 ), (2, 3 )}.
State
(a) the image of 2.
(b) the object of 0.
[2 marks]




2 Given that f : x 3x 4 and g : x x
2
+8 x + 16, find
(a) f
-1
(5)
(b) gf (x)

[4 marks]







3 3 is one of the roots of the quadratic equation 2x
2
+ p x =3
Find
(a) the value of p,
(b) the value of the other root.
[3 marks]








4 The quadratic equation (2x 5)
2
= (p 10) x has two distinct roots.
Find the range of values of p.
[3 marks]







M = {3, 2, 1, 0, 1, 2}
N = {1, 0, 1, 2, 3}
77
Program Cemerlang Bersama SMT Zon Johor Program Cemerlang Bersama SMT Zon Johor Program Cemerlang Bersama SMT Zon Johor Program Cemerlang Bersama SMT Zon Johor
5 Solve
3 3
log (4 ) log ( 1) 1 x x + =
[4 marks]









6
Solve the equation 3 2
x
= .
[3 marks]









7 During the year 2005, a company increased its sales of digital cameras at a constant rate of
200 units per month. Thus the number of digital cameras sold in February was 200 more than
in January, the number of digital cameras sold in March was 200 more than in February, and
this pattern continued month by month throughout the year. Given that the company sold
38,400 units of digital cameras in the year 2005.
Calculate the number of units of digital cameras sold in
(a) January 2005
(b) December 2005
[4 marks]








8 The first three terms of a geometric progression are x + 10, x 2 and x 10 respectively.
Calculate
(a) the value of x.
(b) the sum to infinity of this progression.
[4 marks]










7B
Ja0ap ge|j me:e-t, Jeat|aa, )ee-
9 A point P (8, t) divides the line joining A (4, 1) and B (r, 7) such that 3AP = 2PB. Find the
value of
(a) r
(b) t
[3 marks]











10 The straight lines y = mx + 9, where m is a constant, and 2y = x 2 are perpendicular.
(a) Find the value of m,
(b) Hence or otherwise, find the coordinates of the point of intersection of the lines.
[3 marks]











11

Diagram 1
Diagram 1 shows vectors CB and BA. A, B and C are (0,
h), (5, 3) and (2, 7) respectively.
(a) Express CB in the form of
xi yj +



(b) Given that 5 7 BA i j =


, find the corresponding
value of h.
[2 marks]













79
Program Cemerlang Bersama SMT Zon Johor Program Cemerlang Bersama SMT Zon Johor Program Cemerlang Bersama SMT Zon Johor Program Cemerlang Bersama SMT Zon Johor
12

Diagram 2
Diagram 2 shows a parallelogram ABCD with BED as a
straight line.
Given that 5 AB =

p, BC

= 3q, and 4DE = EB. Express


in terms of p and q :
(a) BD


(b) EC


[4 marks]










13
Given that 2 2 a i j = +


, 2 3 b i j =


and 2 c a b =

. Find
(a) c


(b) unit vector in the direction of c

.
[3 marks]










14 The net value, m in hundreds of Ringgit, of the monthly output of a factory is modeled by
2 3
10 3000
n n
m = where n is the number of working hours a worker put in per month. Calculate
the value of n such that m is maximum.
[3 marks]















A
B
C
D
E

Bo
Ja0ap ge|j me:e-t, Jeat|aa, )ee-
15 The radius of a spherical balloon is increasing at the rate of x cms
-1
. Given that the rate of
change of the volume of the balloon is 25 cm
3
s
-1
when its radius is 5 cm.
Find the value of x.
[
3
4
3
V r = ]
[3 marks]









16

Diagram 3
x and y are related by the equation
m
x ny
x
+ = , where m
and n are constants. A straight line is obtained by plotting
xy against x
2
, as shown in Diagram 3.
Calculate the value of m and of n.
[4 marks]










17
Given that
5
1
( ) 5 g x dx =

, find the value of m if


5
1
[ 2 ( )] 3 mx g x dx m =

.
[3 marks]














xy
x
2

(6, 0)

(12, 2)
B1
Program Cemerlang Bersama SMT Zon Johor Program Cemerlang Bersama SMT Zon Johor Program Cemerlang Bersama SMT Zon Johor Program Cemerlang Bersama SMT Zon Johor
18

Diagram 4
Diagram 4 shows the quadratic curve
y = g(x) and the line PQ. PQ is parallel to
the x-axis.
Given that the curve has a minimum value
at O, the origin, the curve intersects line PQ
at R (b, 4). Given that the area of the shaded
region is
1
3
.
Find 2 ( )
b
b
g x dx


[2 marks]








19 A circle of radius 2.5 cm has a minor sector with an area of 6.25 cm
2
. Calculate
(a) the angle of the sector in radians,
(b) the perimeter of the major sector.
[use 3.142 = ]
[3 marks]










20
Given that x is an acute angle and sin
m
x
n
= , find 1 + tan(90 ) x

in terms of m and n.
[3 marks]















P
Q
y
y = g(x)
R(b,4)

B2
Ja0ap ge|j me:e-t, Jeat|aa, )ee-
21
Solve 3cos 2x + 4 cos x +1 = 0 for 0 360 x


[4 marks]












22 An expedition team consisting of 10 members to be chosen from a group of 4 teachers and 12
students.
(a) Calculate the number of teams that can be formed.
(b) If the team must consist of at least 2 teachers, calculate the numbers of teams that could be
formed.
[3 marks]










23 (a) How many possible arrangements in a row, can all the letters from the word C E R I A
be arranged?
(b) An arrangement in (a) is chosen at random, find the probability that the arrangement will
have both A and E separated.
[4 marks]














B3
Program Cemerlang Bersama SMT Zon Johor Program Cemerlang Bersama SMT Zon Johor Program Cemerlang Bersama SMT Zon Johor Program Cemerlang Bersama SMT Zon Johor
24

Diagram 5
Diagram 5 shows a standard normal distribution
graph.
The probability represented by the area of the shaded
region is 0.803.
(a) Find the value of P( Z > k )
(b) X is a continuous random variable which is
normally distributed with a mean of and a
standard deviation of 2. If the value of X is 85
when the Z-score is k, find the value of .
[3 marks]









25 In a survey, the probability that a family owning one unit of computer is 0.6. N families were
selected at random. The standard deviation of the numbers of family owning one unit of
computer is 1.697.
Find
(a) the value of N
(b) the mean of the numbers of family owning one unit of computer.
[3 marks]














END OF QUESTION PAPER















k k
x
f(x)

B4
Ja0ap ge|j me:e-t, Jeat|aa, )ee-
MODULE MODULE MODULE MODULE 3 33 3
UTUSAN UTUSAN UTUSAN UTUSAN 2008 2008 2008 2008, PAPER , PAPER , PAPER , PAPER 2 22 2

SECTION A
[40 marks]

Answer all questions

1
Solve the simultaneous equations
2
1 1 1
=
x y
and 5 2 = + x y .
[5 marks]

2
The quadratic function q px x x f + + =
2
2 ) ( has a minimum value of 5 when x = 2.
(a) By using the method of completing the squares, find the values of p and q.
[4 marks]
(b) Hence, sketch the graph of q px x x f + + =
2
2 ) (
[3 marks]

3
It is given that
2
3 24 18 x x y = .
(a) Find
dx
dy
and
2
2
dx
y d
,
[3 marks]
(b) Show that x y
dx
y d
x
dx
dy
x 18 2 ) 2 8 ( 3
2
2
2
+ = + .
[3 marks]

4.

Diagram 1 shows a sector OAB of a circle with centre
O and Q is the midpoint of OB.
Given OB = 16 cm, calculate
(a) The value of , in radians,
[3 marks]
(b) The area, in cm
2
, of the shaded region.
[4 marks]


















A
O
B
P
Q

Diagram 1
B5
Program Cemerlang Bersama SMT Zon Johor Program Cemerlang Bersama SMT Zon Johor Program Cemerlang Bersama SMT Zon Johor Program Cemerlang Bersama SMT Zon Johor
5 The histogram in Diagram 2 shows the score distribution of 40 students in a test.

Diagram 2

Score Frequency Cumulative
Frequency
80 89 4
90 99




Table 1


(a) Based on the histogram, copy and complete Table 1.
[2 marks]
(b) Calculate the interquartile range.
[5 marks]


6

In Diagram 3, OPQR is a parallelogram. OTR, PTS
and QRS are the straight lines.
It is given that x OP 8 = , y OR 12 = and OT : TR =
3 : 1.
(a) Express PT in terms of x and y.
(b) Given that the area of PQS = 35 units
2
and the
perpendicular distance from point P to QS is 3
units, find |c|.
[8 marks]


SECTION B
[40 marks]
Answer four questions in this section

7
Use the graph paper to answer this question.
Table 2 shows the values of two variables, x and y, obtained from and experiment. The
variables x and y are related by the equation ) ( q x px y = , where p and q are constants.
x 1 2 3 4 5 6
y 1.5 9 22.5 42 67.5 99
Table 2
(a) Plot
x
y
against x, using the scale of 2 cm to 1 unit on the x-axis and 2cm to 2 units on the
x
y
-axis.
[5 marks]
(b) Use your graph in 7(a) to find the value of
i) p,
ii) q.
[5 marks]
O P
Q
R
S
T
Diagram 3

BB
Ja0ap ge|j me:e-t, Jeat|aa, )ee-
8

In Diagram 4, the straight line MN is the tangent to the
curve
2
) 1 (
2

=
x
y at point B. The line NA is parallel to
y-axis.
Find
(a) the equation of the tangent MN and hence, find the
coordinates of point N.
[3 marks]
(b) the area of the shaded region,
[4 marks]
(c) the volume generated, in terms of , of the region
bounded by the curve, the line NA and x = 2 is
revolved through 360 about the x-axis.
[3 marks]


9 In Diagram 5, the straight line AB is perpendicular to the straight line BC.

It is given that the equation of the straight line AB is 2y + x 7 = 0.
(a) Find
i) the equation of the straight line BC,
ii) the coordinates of B.
[4 marks]
(b) The straight line CB is extended to a point D such that CB : BD = 2 : 3. Find the
coordinates of D.
[3 marks]
(c) A point P moves such that its distance from point C is always 4 units. Find the equation of
the locus of P.
[3 marks]

10 (a) Prove that cosec 2A cot 2A = tan A.
[3 marks]
(b) (i) Sketch the graph of y = |cos x| + 1 for 0 < x < 2
[4 marks]
(ii) Hence, using the same axes, sketch a suitable graph to find the number of solutions
for the equation 2 cos 2 =
x
x

for 0 < x < 2. State the number of solutions.
[3 marks]

M
N

B(2, 2)
2
) 1 (
2

=
x
y
A
y
x
Diagram 4
O
C(1, 11)
x + 2y 7 = 0
A
B
0
y
x
Diagram 5
B7
Program Cemerlang Bersama SMT Zon Johor Program Cemerlang Bersama SMT Zon Johor Program Cemerlang Bersama SMT Zon Johor Program Cemerlang Bersama SMT Zon Johor
11 For this question, give your answer correct to three significant figures.
(a) Ahmad is a golf player. The probability that he successfully hits a ball into a hole is p.
i) Ahmad makes 50 hits. Find the possible values of p if the variance of the number of
successes is 8.
ii) Ahmad makes 7 hits. Using the larger value of p obtained in part (a) i) find the
probability that he hits a ball into a hole is at least 3.
[5marks]
(b) The mass of a tablet produced by a factory is normally distributed with mean mg and a
standard deviation 0.5 mg.
i) Find the value of if 15.87% of the tablets chosen at random have a mass of more
than 7.5 mg.
ii) Find the number of tablets that have a mass between 6.4 mg and 7.8 mg if 2000 tablets
are produced in a certain period.
[5 marks]

SECTION C
[20 marks]

Answer two questions in this section

12

Diagram 6 shows a pyramid with horizontal base ABC
and vertical height VC.
Given the ACB = 80, VAC = 45, AB = 100 cm and
VC = 15 cm.
Find
(a) the length, in cm, of AC and AV.
[3 marks]
(b) the BAC,
[4 marks]
(c) the area, in cm
2
, of triangle ABC.
[3 marks]

13 A factory makes two types of plastic containers, cylindrical and cubic shape. Two machines P
and Q are used to produce the plastic containers. Table 4 shows the time taken by each
machine to make the cylindrical and cubic shape containers.

Time taken (minutes)
Machine Cylindrical container Cubic container
P 6 4
Q 2 7
Table 4

The factory produces x cylindrical container and y cubic containers per day. The production
of the plastic containers is based on the following constraints.
I The total time of usage of machine P cannot be more than 10 hours.
II The total time of usage of machine Q must be at least 280 minutes.
III The number of cubic containers produced must exceed twice the number of
cylindrical containers produced.

Diagram 6
C
A
B
V

BB
Ja0ap ge|j me:e-t, Jeat|aa, )ee-
(a) Write three inequalities, other than x > 0 and y > 0 which satisfy the constraints.
[3 marks]
(b) Using the scale of 2 cm to 20 containers on both axes, construct and shade the region R
which satisfies all the constraints.
[4 marks]
(c) Using your graph from (b), find
i) the maximum number of cylindrical containers if the number of both cylindrical and
cubic containers produced are the same.
ii) the maximum profit that can be obtained if the profit from each cylindrical containers
and each cubic container are RM30 and RM 40 respectively.
[3 marks]

14 A particle P moves along a straight line beginning from a fixed point O. The velocity of
particle P, v ms
-1
, is given by v = 18 3t t
2
, where t is the time in second after leaving O.
Assume the positive direction of the motion is to the right, find
(a) the value of t when particle P changes direction.
[3 marks]
(b) the maximum displacement of particle P to the right of O.
[4 marks]
(c) the total distance traveled, in m, by particle P in the first 4 seconds.
[3 marks]

15 Table 5 shows the price indices in the year 2007 based to the year 2006, of four different
materials A, B, C and D, in the production of a type of a shampoo. It also includes the
division of the usages of the materials in the production of the shampoo.

Material
Price Index 2007
(2006 = 100)
Weightage
A 125 4
B 120 p
C 80 5
D 150 p + 3
TABLE 5

(a) If the price of material A is RM 50 in the year 2007, calculate its price in 2006.
[2 marks]
(b) If the composite index for the year 2007 based to the year 2006 is 120, find the
value of p.
[2 marks]
(c) Find the price of the shampoo in 2007 if its price in 2006 was RM 15.00
[2 marks]
(d) Given that the price of material C is estimated to increase by 15 % from the year
2007 to 2008, while the others remain unchanged. Calculate the composite index
of the shampoo in the year 2008, based on the year 2006.
[4 marks]

END OF QUESTION PAPER






B9
Program Cemerlang Bersama SMT Zon Johor Program Cemerlang Bersama SMT Zon Johor Program Cemerlang Bersama SMT Zon Johor Program Cemerlang Bersama SMT Zon Johor
MODULE MODULE MODULE MODULE 4 44 4
JPTECH 2007 JPTECH 2007 JPTECH 2007 JPTECH 2007, PAPER , PAPER , PAPER , PAPER 2 22 2

SECTION A
[40 marks]
Answer all questions

1
Solve the simultaneous equations 1 4 = x y and x y 16 25
2
= .
[5 marks]

2
Given a quadratic equation 1 6 2 ) (
2
+ = x x x f .
(a) Express f(x) in the form of a(x + p)
2
+ q, where a, p and q are constants and state the
maximum or minimum value of f(x) and the corresponding value of x.
[4 marks]
(b) Hence, sketch the graph of function f(x).
[2 marks]

3 A set of data consists of four numbers has variance 0.25 and the sum of the squares of the
numbers is 10.
(a) Find the mean of the numbers in the set.
[3 marks]
(b) If each of number in the set is added with a constant p and then multiplied by 6, the new
mean and variance are 12 and k respectively where k is a constant.
Find the value of p and of k.
[4 marks]

4 Diagram 1 shows two straight lines, AB and BC that are perpendicular to each other at point B.
Points A and B are on the x-axis and y-axis respectively.

(a) The equation of the line AB is 4y 3x 16 = 0.
Find the equation of CB.
[3 marks]
(b) If CB is extended until it intersects the x-axis at point D such that CB : BD = 1 : 3, find the
coordinates of point C.
[3 marks]

x
y
0 A
B
4y 3x 16 =0
C
Diagram 1

9o
Ja0ap ge|j me:e-t, Jeat|aa, )ee-
5 (a) Prove that tan
2
B - sin
2
B = tan
2
B sin
2
B .
[3 marks]
(b) (i) Sketch the graph x y 2 sin
2
1
+ = for 2 0 x
(ii) Hence, using the same axes, draw a suitable straight line to find the number of
solutions to the equation
2 2
1
cos sin
x
x x = + for 2 0 x . State the number of solutions.
[5 marks]

6 Diagram 2 shows a part of a series of squares with ABCD as the first square with sides 24 cm,
AEFG the second square, AHIJ the third square and so on. Each square is drawn in such a way
that the length of each side is half that of the previous square.

(a) Find the area of the fifth square in the series.
[3 marks]
(b) Estimate the total area of all the squares that can be drawn in this manner.
[2 marks]
(c) Find which square that has its length of sides less than 0.05 cm for the first time.
[3 marks]


SECTION B
[40 marks]
Answer four questions in this section

7 Use the graph paper to answer this question.
Table 1 shows the values of two variables, x and y obtained from an experiment. Variables x
and y are related by the equation qy = px
2
+ x where p and q are constants.

x 1 2 4 7 8 9
y 0.2 1.2 5.6 18.2 24 30.6
Tabel 1
(a) Plot
x
y
against x by using a scale of 2 cm to 1 unit on the x-axis and 2 cm to 0.5 unit on
the
x
y
-axis. Hence, draw the line of best fit.
[4 marks]
(b) Use your graph fiom 7(a) to find the value of
(i) p and q,
(ii) y when
x
y
= 2.15.
[6 marks]
A H E B
I J
G
D
C
F
24 cm
24 cm
Diagram 2
91
Program Cemerlang Bersama SMT Zon Johor Program Cemerlang Bersama SMT Zon Johor Program Cemerlang Bersama SMT Zon Johor Program Cemerlang Bersama SMT Zon Johor
8 Diagram 3 shows a triangle OPQ.

T is the midpoint of OQ and the straight line OS intersects the straight line PT at U.
Given that p OP 8 = , q OQ 6 = and PQ PS
4
1
=
(a) Express in terms of p and / or q ,
(i) PQ
(ii) OS ,
(iii) PT
[3 marks]
(b) (i) Given that PT h PU = , express PU in terms of h, p and q.
(ii) Given that OS k OU = , express OU in terms of k, p and q .
[2 marks]
(c) Using PU and OU from 8(b), find the values of h and k.
[5 marks]

9 Diagram 4 shows sector OAB and sector OQP both with centre O.

The point A lies on OP, point R lies on OQ and AR is perpendicular to OQ. The length of OA
= OB = 9 cm, POQ = rad
5

and AOB = 2POQ. It is given that OA : AP = 3 : 2.


Using = 3.142, calculate
(a) the length, in cm, of OR,
[l marks]
(b) the perimeter of the shaded regions,
[5 marks]
(c) the area of the shaded regions.
[4 marks]

O R
Q
P
A
B
9 cm
rad
5


Diagram 4
P
O
T
Q
S
U
Diagram 3

92
Ja0ap ge|j me:e-t, Jeat|aa, )ee-
10 Diagram 5 shows the straight line y = x intersects the curve y = x(x 4) at point B.

Find
(a) the coordinates of point B,
[l mark]
(b) the equation of normal to the curve at point (3, 1),
[4 marks]
(c) the volume generated when the shaded region is revolved 360 about the x-axis.
[5 marks]

11 (a) A study carried out in a technical school; found that 6 out of 8 students attended
Additional Mathematics tuition.
i) If 7 students are chosen at random, calculate the probability that at least 5 students
attended Additional Mathematics tuition.
[3 marks]
ii) If the standard deviation of the number of students who attended the tuition is 12, find
the total number of students in the school.
[2 marks]
(b) Marks obtained by a group of students in an examination are normally distributed with a
mean of 48 marks and standard deviation of 10 marks.
i) If a student is selected at random, calculate the probability that the student's mark is
more than 60.
[2 marks]
ii) If 30% of the students fail in the examination, estimate the minimum mark required to
pass the examination.
[3 marks]






















y
x
y = x(x 4)
4 O
y = x
Diagram 5
B
93
Program Cemerlang Bersama SMT Zon Johor Program Cemerlang Bersama SMT Zon Johor Program Cemerlang Bersama SMT Zon Johor Program Cemerlang Bersama SMT Zon Johor
SECTION C
[20 marks]
Answer two questions in this section

12 Table 2 shows the price indices of the usage of four items, A, B, C and D which are the main
ingredients in making a particular kind of cake in the year 2002 based on the year 2000 and
the changes in the price indices from the year 2002 to the year 2005 with their respective
weightage.

Item
Price Index in
2002
Changes of Price Index
from 2002 to 2005 Weightage
A k Unchanged 5
B 110 Increased 10% 2
C 120 Decreased 5% 3
D 115 Unchanged 10
Table 2

(a) Find the price of the item B in the year 2002 if the price in the year 2000 is RM5.00.
[2 marks]
(b) If the composite index for the year 2002 is 116.5, find the value of k.
[3 marks]
(c) Calculate the composite index for the year 2005 based on the year 2000. Give your answer
correct to the nearest integer.
[3 marks]
(d) If Encik Azman spent RM85 to make a certain quantity of the cake in the year 2002, how
much would he spend for making the same quantity of cakes in the year 2005?
[2 marks]

13 Diagram 6 shows a quadrilateral ABCD where BCD is an acute angle and the area of
triangle BCD is 15 cm
2
.

(a) Find the length, in cm, of BD.
[4 marks]
(b) Calculate the two possible values of BDA.
[3 marks]
(c) Using the acute angle of BDA, calculate, in cm
2
, area of the quadrilateral ABCD.
[3 marks]


94
Ja0ap ge|j me:e-t, Jeat|aa, )ee-
14 Diagram 7 shows two fixed point 0 and A which is 15 m apart.

A particle P moves along a straight line from point A. Its velocity, v ms
-1
is given by
v = 8 2t, where t is the time, in seconds, after leaving point A.
Find
(a) the initial velocity and acceleration,
[2 marks]
(b) the displacement of the particle P from 0 when
(i) t = 2 s ,
(ii) t = 10 s,
[3 marks]
(c) the time taken when particle P reverses its direction of motion,
[2 marks]
(d) the distance travelled by P from t = 2 s to t = 10 s.
[3 marks]
[Assume that the motion to the right is positive]

15 Use the graph paper provided to answer this question.
A trader intends to spend RM4 000 to buy x pairs of leather shoes cost RM 90 each and y
pairs of canvas shoes cost RM50 each. The number of pairs of canvas shoes must exceed the
number of pairs of leather shoes by 10 or more. The number of pairs of leather shoes must be
at least 10% that of canvas shoes.

(a) Write down three inequalities, other than x > 0 and y > 0, which satisfy all the above
constraints.
[3 marks]
(b) Using a scale of 2 cm to 5 pairs of leather shoes on the x-axis and 2 cm to 10 pairs of
canvas shoes on the y-axis, draw the graphs for the three inequalities.
Hence, shade the region R that satisfies all the above constraints.
[3 marks]
(c) By using your graph from 15(b), answer the following questions.
i) Find the maximum number of pairs of shoes that can be bought by the trader if the
number of pairs of leather shoes is 10% that of the canvas shoes.
ii) During the sale, the trader sold each pair of leather and canvas shoes at RM114 and
RM62 respectively. Calculate the maximum profit gained by the trader for selling all
his shoes.
[4 marks]

END OF QUESTION PAPER










95
Program Cemerlang Bersama SMT Zon Johor Program Cemerlang Bersama SMT Zon Johor Program Cemerlang Bersama SMT Zon Johor Program Cemerlang Bersama SMT Zon Johor


ANSWERS FOR MODULES ANSWERS FOR MODULES ANSWERS FOR MODULES ANSWERS FOR MODULES

9B
Ja0ap ge|j me:e-t, Jeat|aa, )ee-



1 (a) {2, 0}
(b) {7, 1}

2 (a) x 4
(b) 49 14x + x
2


3 0.886 or 3.386

4 x 2 x 5

5
6
5
= a

6
35
18
= x

7 y = 64x

8 T
5
= 42

9
2
1
7 , 3 = = a r

10
330
53


11
2
2
q
p =

12
3 ,
2
3
= = k h

13 (a) 2i 6j
(b)
5
4 3 j i +


14 m = 1, n = 2

15 16.35 cm
2


16 123.69, 303.69

17
2
2
) 2 5 (
8 30 6
t
t t

+ +


MODULE MODULE MODULE MODULE 1 11 1 (PAPER 1) (PAPER 1) (PAPER 1) (PAPER 1)
18 (2, 20), (2, 44)

19 9

20
5
4


21
3
5


22 (a) 201,600
(b) 17,280

23 (a) 38,760
(b) 3,060

24
7
2


97
Program Cemerlang Bersama SMT Zon Johor Program Cemerlang Bersama SMT Zon Johor Program Cemerlang Bersama SMT Zon Johor Program Cemerlang Bersama SMT Zon Johor



1 (a) 3
(b) 1

2 (a) 3
(b) 9x
2


3 (a) 5
(b)
2
1


4 p < 30 , p > 10

5
2
3


6 0.6309

7 (a) 2100
(b) 4300

8 (a) 26
(b) 108

9
r = 14, t =
17
5


10 (a) 2
(b) (4, 1)

11 3 4 i j




12 (a) 5p+3q
(b) 4p+
3
5
q

13 (a) 10
(b)
1
( 2 )
5
i j +

14 200

15 1
4


16 m = 6 , n = 3

MODULE 2 (PAPER 1) MODULE 2 (PAPER 1) MODULE 2 (PAPER 1) MODULE 2 (PAPER 1)
17 2
3


18 4
3


19 (a) 2 =
(b) 15.71

20
2
2
2
( )
n n
or
m m


21
70.53 ,180 , 289.47



22 (a) 8008
(b) 7062

23 (a) 120
(b)
3
5


24 (a) 0.0985
(b) 82.42

25 (a) 12
(b) 7.2
















9B
Ja0ap ge|j me:e-t, Jeat|aa, )ee-

1
x = 2,
2
5
, y = 1, 10

2 (a) p = 8, q = 13
(b)

MODULE MODULE MODULE MODULE 3 33 3 (PAPER (PAPER (PAPER (PAPER 2 22 2) )) )
3
(a) x
dx
dy
6 18 = , 6
2
2
=
dx
y d

(b) 2y + 18x

4 (a) 0.7854 rad.
(b) 18.27 cm
2


5 (a)
Score Freq Cum. Freq.
80 89 4 4
90 99 7 11
100 109 9 20
110 119 8 28
120 129 7 35
130 139 5 40
(b) 24.29

6 (a) 8x + 9y
(b) 2.188 units

7 (a)

7
pq px
x
y
=

(b) (i) p = 3, (ii)
2
1
= q

8 (a) y = 4x + 10, N(2.5, 0)

(b)
2
6
1
unit (c)
81
76
unit
3

9 (a) y = 2x 9 , (ii) B(5, 1)
(b) D(14, 19)
(c) x
2
+ y
2
+ 2x + 22y + 106 = 0

10

Number of solutions = 2

11
(a)(i)
5
1
5
4
or p = , (ii) 0.995
(b)(i) = 7, (ii) 1660

12 (a) AC = 15 cm, AV = 21.21 cm
(b) BAC = 91.5
(c) 747.7 cm
2


13 (a) I: 3x + 2y < 300
II: 2x + 7y < 280
III: y < 2x
(c)(i) 60, (ii) RM 4690

99
Program Cemerlang Bersama SMT Zon Johor Program Cemerlang Bersama SMT Zon Johor Program Cemerlang Bersama SMT Zon Johor Program Cemerlang Bersama SMT Zon Johor


3 (a) Mean = 1.5
(b) p = 0.5, k = 9

4 (a) 3y = 4x + 12

(b)

3
16
, 1 C

5
14 (a) t = 3
(b) Max. displacement 31.5 m

(c) Total distance traveled
3
1
36 m

15 (a) RM 40
(b) p = 3
(c) RM 18
(d) 123.33

MODULE MODULE MODULE MODULE 4 44 4 (PAPER (PAPER (PAPER (PAPER 2 22 2) )) )
1
1 ,
2
3
, 3 , 7 = = x y

2
(a)
2
7
2
3
2 ) (
2

= x x f
Min value is
2
7
when x =
2
3


The number of solutions = 3

6
(a)
4
9

(b) 768
(c) n = 10

7 (a) Y-intercept = 0.2

(b)(i)
q q
px
x
y 1
+

=
q = 5, p = 2
(ii) y = 12.685


1oo
Ja0ap ge|j me:e-t, Jeat|aa, )ee-

8 (a)(i) -8p + 6q

(ii) 6p+
2
3
q
(iii) -8p + 3q
(b) (i) -8hp+3hq

(ii) 6kp+
2
3
kq

(c)
5
4
,
5
2
= = k h

9 (a) 7.281 cm
(b) 57.743 cm
(c) 102.321 cm
2


10 (a) B(5, 5)
(b) 2y = x + 1
(c) 34.133 or 107.23

11 (a)(i) 0.7564
(ii) n = 768
(b)(i) 0.1151
(ii) Min. passing mark = 42.76

12 (a) RM5.50 (b) k = 120
(c) 117 (d) x = RM85.36

13 (a) BD = 5.12 cm
(b) BDA = 5123, 12837
(c) 35.249 cm
2


14 (a) v = 8, a = 2
(b)(i) 27 m, (ii) 5 m
(c) t = 4
(d) 40 m

15 (a) I 9x + 5y 400
II y x + 10
III y x
10
1

(c)(i) Max. pairs of shoes = 66
(ii) Max. profit = RM 1020

15(b)
1o1
Program Cemerlang Bersama SMT Zon Johor Program Cemerlang Bersama SMT Zon Johor Program Cemerlang Bersama SMT Zon Johor Program Cemerlang Bersama SMT Zon Johor


COLLECTION OF COLLECTION OF COLLECTION OF COLLECTION OF
SPM TRIAL 2008 SPM TRIAL 2008 SPM TRIAL 2008 SPM TRIAL 2008
1 11 1. .. . SELANGOR SELANGOR SELANGOR SELANGOR
2 22 2. .. . KUALA LUMPUR KUALA LUMPUR KUALA LUMPUR KUALA LUMPUR
3 33 3. .. . SARAWAK SARAWAK SARAWAK SARAWAK
4 44 4. .. . PERLIS PERLIS PERLIS PERLIS
5 55 5. .. . ANSWERS ANSWERS ANSWERS ANSWERS

1o2
Ja0ap ge|j me:e-t, Jeat|aa, )ee-
1o3
Program Cemerlang Bersama SMT Zon Johor Program Cemerlang Bersama SMT Zon Johor Program Cemerlang Bersama SMT Zon Johor Program Cemerlang Bersama SMT Zon Johor

1o4
Ja0ap ge|j me:e-t, Jeat|aa, )ee-
1o5
Program Cemerlang Bersama SMT Zon Johor Program Cemerlang Bersama SMT Zon Johor Program Cemerlang Bersama SMT Zon Johor Program Cemerlang Bersama SMT Zon Johor

1oB
Ja0ap ge|j me:e-t, Jeat|aa, )ee-
1o7
Program Cemerlang Bersama SMT Zon Johor Program Cemerlang Bersama SMT Zon Johor Program Cemerlang Bersama SMT Zon Johor Program Cemerlang Bersama SMT Zon Johor

1oB
Ja0ap ge|j me:e-t, Jeat|aa, )ee-
1o9
Program Cemerlang Bersama SMT Zon Johor Program Cemerlang Bersama SMT Zon Johor Program Cemerlang Bersama SMT Zon Johor Program Cemerlang Bersama SMT Zon Johor

11o
Ja0ap ge|j me:e-t, Jeat|aa, )ee-
111
Program Cemerlang Bersama SMT Zon Johor Program Cemerlang Bersama SMT Zon Johor Program Cemerlang Bersama SMT Zon Johor Program Cemerlang Bersama SMT Zon Johor

112
Ja0ap ge|j me:e-t, Jeat|aa, )ee-
113
Program Cemerlang Bersama SMT Zon Johor Program Cemerlang Bersama SMT Zon Johor Program Cemerlang Bersama SMT Zon Johor Program Cemerlang Bersama SMT Zon Johor

114
Ja0ap ge|j me:e-t, Jeat|aa, )ee-
115
Program Cemerlang Bersama SMT Zon Johor Program Cemerlang Bersama SMT Zon Johor Program Cemerlang Bersama SMT Zon Johor Program Cemerlang Bersama SMT Zon Johor

11B
Ja0ap ge|j me:e-t, Jeat|aa, )ee-
117
Program Cemerlang Bersama SMT Zon Johor Program Cemerlang Bersama SMT Zon Johor Program Cemerlang Bersama SMT Zon Johor Program Cemerlang Bersama SMT Zon Johor

11B
Ja0ap ge|j me:e-t, Jeat|aa, )ee-
119
Program Cemerlang Bersama SMT Zon Johor Program Cemerlang Bersama SMT Zon Johor Program Cemerlang Bersama SMT Zon Johor Program Cemerlang Bersama SMT Zon Johor

12o
Ja0ap ge|j me:e-t, Jeat|aa, )ee-
121
Program Cemerlang Bersama SMT Zon Johor Program Cemerlang Bersama SMT Zon Johor Program Cemerlang Bersama SMT Zon Johor Program Cemerlang Bersama SMT Zon Johor

122
Ja0ap ge|j me:e-t, Jeat|aa, )ee-
123
Program Cemerlang Bersama SMT Zon Johor Program Cemerlang Bersama SMT Zon Johor Program Cemerlang Bersama SMT Zon Johor Program Cemerlang Bersama SMT Zon Johor

124
Ja0ap ge|j me:e-t, Jeat|aa, )ee-
125
Program Cemerlang Bersama SMT Zon Johor Program Cemerlang Bersama SMT Zon Johor Program Cemerlang Bersama SMT Zon Johor Program Cemerlang Bersama SMT Zon Johor

12B
Ja0ap ge|j me:e-t, Jeat|aa, )ee-
127
Program Cemerlang Bersama SMT Zon Johor Program Cemerlang Bersama SMT Zon Johor Program Cemerlang Bersama SMT Zon Johor Program Cemerlang Bersama SMT Zon Johor

12B
Ja0ap ge|j me:e-t, Jeat|aa, )ee-
129
Program Cemerlang Bersama SMT Zon Johor Program Cemerlang Bersama SMT Zon Johor Program Cemerlang Bersama SMT Zon Johor Program Cemerlang Bersama SMT Zon Johor

13o
Ja0ap ge|j me:e-t, Jeat|aa, )ee-
131
Program Cemerlang Bersama SMT Zon Johor Program Cemerlang Bersama SMT Zon Johor Program Cemerlang Bersama SMT Zon Johor Program Cemerlang Bersama SMT Zon Johor

132
Ja0ap ge|j me:e-t, Jeat|aa, )ee-
133
Program Cemerlang Bersama SMT Zon Johor Program Cemerlang Bersama SMT Zon Johor Program Cemerlang Bersama SMT Zon Johor Program Cemerlang Bersama SMT Zon Johor

134
Ja0ap ge|j me:e-t, Jeat|aa, )ee-
135
Program Cemerlang Bersama SMT Zon Johor Program Cemerlang Bersama SMT Zon Johor Program Cemerlang Bersama SMT Zon Johor Program Cemerlang Bersama SMT Zon Johor

13B
Ja0ap ge|j me:e-t, Jeat|aa, )ee-
137
Program Cemerlang Bersama SMT Zon Johor Program Cemerlang Bersama SMT Zon Johor Program Cemerlang Bersama SMT Zon Johor Program Cemerlang Bersama SMT Zon Johor

13B
Ja0ap ge|j me:e-t, Jeat|aa, )ee-
139
Program Cemerlang Bersama SMT Zon Johor Program Cemerlang Bersama SMT Zon Johor Program Cemerlang Bersama SMT Zon Johor Program Cemerlang Bersama SMT Zon Johor

14o
Ja0ap ge|j me:e-t, Jeat|aa, )ee-
141
Program Cemerlang Bersama SMT Zon Johor Program Cemerlang Bersama SMT Zon Johor Program Cemerlang Bersama SMT Zon Johor Program Cemerlang Bersama SMT Zon Johor

142
Ja0ap ge|j me:e-t, Jeat|aa, )ee-
143
Program Cemerlang Bersama SMT Zon Johor Program Cemerlang Bersama SMT Zon Johor Program Cemerlang Bersama SMT Zon Johor Program Cemerlang Bersama SMT Zon Johor

144
Ja0ap ge|j me:e-t, Jeat|aa, )ee-
145
Program Cemerlang Bersama SMT Zon Johor Program Cemerlang Bersama SMT Zon Johor Program Cemerlang Bersama SMT Zon Johor Program Cemerlang Bersama SMT Zon Johor

14B
Ja0ap ge|j me:e-t, Jeat|aa, )ee-
147
Program Cemerlang Bersama SMT Zon Johor Program Cemerlang Bersama SMT Zon Johor Program Cemerlang Bersama SMT Zon Johor Program Cemerlang Bersama SMT Zon Johor

14B
Ja0ap ge|j me:e-t, Jeat|aa, )ee-
149
Program Cemerlang Bersama SMT Zon Johor Program Cemerlang Bersama SMT Zon Johor Program Cemerlang Bersama SMT Zon Johor Program Cemerlang Bersama SMT Zon Johor

15o
Ja0ap ge|j me:e-t, Jeat|aa, )ee-
151
Program Cemerlang Bersama SMT Zon Johor Program Cemerlang Bersama SMT Zon Johor Program Cemerlang Bersama SMT Zon Johor Program Cemerlang Bersama SMT Zon Johor

152
Ja0ap ge|j me:e-t, Jeat|aa, )ee-
153
Program Cemerlang Bersama SMT Zon Johor Program Cemerlang Bersama SMT Zon Johor Program Cemerlang Bersama SMT Zon Johor Program Cemerlang Bersama SMT Zon Johor

154
Ja0ap ge|j me:e-t, Jeat|aa, )ee-
155
Program Cemerlang Bersama SMT Zon Johor Program Cemerlang Bersama SMT Zon Johor Program Cemerlang Bersama SMT Zon Johor Program Cemerlang Bersama SMT Zon Johor

15B
Ja0ap ge|j me:e-t, Jeat|aa, )ee-
157
Program Cemerlang Bersama SMT Zon Johor Program Cemerlang Bersama SMT Zon Johor Program Cemerlang Bersama SMT Zon Johor Program Cemerlang Bersama SMT Zon Johor

15B
Ja0ap ge|j me:e-t, Jeat|aa, )ee-
159
Program Cemerlang Bersama SMT Zon Johor Program Cemerlang Bersama SMT Zon Johor Program Cemerlang Bersama SMT Zon Johor Program Cemerlang Bersama SMT Zon Johor

1Bo
Ja0ap ge|j me:e-t, Jeat|aa, )ee-
1B1
Program Cemerlang Bersama SMT Zon Johor Program Cemerlang Bersama SMT Zon Johor Program Cemerlang Bersama SMT Zon Johor Program Cemerlang Bersama SMT Zon Johor

1B2
Ja0ap ge|j me:e-t, Jeat|aa, )ee-
1B3
Program Cemerlang Bersama SMT Zon Johor Program Cemerlang Bersama SMT Zon Johor Program Cemerlang Bersama SMT Zon Johor Program Cemerlang Bersama SMT Zon Johor

1B4
Ja0ap ge|j me:e-t, Jeat|aa, )ee-
1B5
Program Cemerlang Bersama SMT Zon Johor Program Cemerlang Bersama SMT Zon Johor Program Cemerlang Bersama SMT Zon Johor Program Cemerlang Bersama SMT Zon Johor

1BB
Ja0ap ge|j me:e-t, Jeat|aa, )ee-
1B7
Program Cemerlang Bersama SMT Zon Johor Program Cemerlang Bersama SMT Zon Johor Program Cemerlang Bersama SMT Zon Johor Program Cemerlang Bersama SMT Zon Johor

1BB
Ja0ap ge|j me:e-t, Jeat|aa, )ee-
1B9
Program Cemerlang Bersama SMT Zon Johor Program Cemerlang Bersama SMT Zon Johor Program Cemerlang Bersama SMT Zon Johor Program Cemerlang Bersama SMT Zon Johor

17o
Ja0ap ge|j me:e-t, Jeat|aa, )ee-
171
Program Cemerlang Bersama SMT Zon Johor Program Cemerlang Bersama SMT Zon Johor Program Cemerlang Bersama SMT Zon Johor Program Cemerlang Bersama SMT Zon Johor

172
Ja0ap ge|j me:e-t, Jeat|aa, )ee-
173
Program Cemerlang Bersama SMT Zon Johor Program Cemerlang Bersama SMT Zon Johor Program Cemerlang Bersama SMT Zon Johor Program Cemerlang Bersama SMT Zon Johor

174
Ja0ap ge|j me:e-t, Jeat|aa, )ee-
175
Program Cemerlang Bersama SMT Zon Johor Program Cemerlang Bersama SMT Zon Johor Program Cemerlang Bersama SMT Zon Johor Program Cemerlang Bersama SMT Zon Johor

17B
Ja0ap ge|j me:e-t, Jeat|aa, )ee-
177
Program Cemerlang Bersama SMT Zon Johor Program Cemerlang Bersama SMT Zon Johor Program Cemerlang Bersama SMT Zon Johor Program Cemerlang Bersama SMT Zon Johor

17B
Ja0ap ge|j me:e-t, Jeat|aa, )ee-
179
Program Cemerlang Bersama SMT Zon Johor Program Cemerlang Bersama SMT Zon Johor Program Cemerlang Bersama SMT Zon Johor Program Cemerlang Bersama SMT Zon Johor

1Bo
Ja0ap ge|j me:e-t, Jeat|aa, )ee-
1B1
Program Cemerlang Bersama SMT Zon Johor Program Cemerlang Bersama SMT Zon Johor Program Cemerlang Bersama SMT Zon Johor Program Cemerlang Bersama SMT Zon Johor

1B2
Ja0ap ge|j me:e-t, Jeat|aa, )ee-

Anda mungkin juga menyukai